AIIMS PG May 2019

 

Kindly Choose the Subject from the Above Menu Bar to show the questions of that particular subject from AIIMS PG May 2019.

Anatomy

Question.1

Finger with two dorsal interossei attached?

AIIMS PG May-2019
A. Little
B. Index
C. Middle
D. Ring
Correct Ans: C
Explanation

Ans.C. Middle finger

  • 1st: largest; lateral side of index finger
  • 2nd and 3rd: both attach to either side of 3rd (middle) finger
  • 4th: medial side of 4th (ring) finger


Question.2

Arrange the following in sequence from Lateral to medial

AIIMS PG May-2019
A. Inferior colliculus
B. Medial geniculate body
C. Cochlear nucleus and Lateral lemniscus
D. All of the above
Correct Ans: D
Explanation

Answer: c->d->a->b

Auditory pathway: SC-SLIM-41,42

  • S– Spiral ganglia (cochlea)
  • C– Cochlear nuclei (ponto-medullary junction)
  • S-Superior olivary nucleus (pons)
  • L-Lateral lemniscus (brain stem)
  • I-Inferior colliculus (mid-brain)
  • M-Medial geniculate body (meta-thalamus)
  • 41,42 – Brodmann area (cerebral cortex)

Question.3

Which is not a branch of the external carotid artery supplying nasal septum?

AIIMS PG May-2019
A. Sphenopalatine
B. greater palatine
C. superior labial
D. anterior ethmoidal
Correct Ans: D
Explanation

Ans. D. anterior ethmoidal

The nasal septum also derives its blood supply from :

  • Branches from the external carotid artery are the sphenopalatine artery, the greater palatine artery, the superior labial artery, and the angular artery.
  • The main branches from the interior carotid are the anterior ethmoidal artery, and the posterior ethmoidal artery that supplies the septum, and these derive from the ophthalmic artery.

Question.4

Mark true or false among the following:

Content ‘s of Carotid Sheath are

AIIMS PG May-2019
A. Internal jugular vein i) true ii) false
B. Cervical sympathetic trunk i) true ii) false
C. Vagus nerve i) true ii) false
D. Internal carotid artery i) true ii) false
E. Deep cervical lymph nodes i) true ii) false
Correct Ans: All:Internal jugular vein i) true ii) false, Cervical sympathetic trunk i) true ii) false, Vagus nerve i) true ii) false, Internal carotid artery i) true ii) false, Deep cervical lymph nodes i) true ii) false
Explanation

Ans.A. i) B. ii)  C. i) D. i) E. i)

The carotid sheath also is a tubular fascial investment that extends superiorly between the cranial base and inferiorly to the root of the neck. The carotid sheath contains the common and internal carotid arteries, internal jugular vein, and vagus nerve [cranial nerve (CN) X]. In addition, the carotid sheath contains deep cervical lymph nodes, sympathetic fibers, and the carotid sinus nerve. The cervical sympathetic trunk lies behind the sheath but is not included within it.


Question.5

Common carotid artery divides at the level of?

AIIMS PG May-2019
A. Hyoid bone
B. Cricoid cartilage
C. Superior border of thyroid cartilage
D. Inferior border of thyroid cartilage
Correct Ans: C
Explanation

Ans. C. Superior border of thyroid cartilage

Common carotid artery divides into internal and external branches at the level of fourth cervical vertebra which also corresponds to the upper border of thyroid cartilage.

Branches of External carotid artery are:

  • Superior thyroid artery
  • Lingual artery
  • Facial artery
  • Ascending pharyngeal artery
  • Occipital artery
  • Posterior auricular artery
  • Maxillary artery
  • Superficial temporal artery

Branches of Internal carotid artery: 

  • Cervical portion has no branches

Branches from other portions are: 

  • Tympanic branch
  • Artery of the pterygoid canal
  • Cavernous branches
  • Hypophyseal branches
  • Anterior meningeal branch
  • Ophthalmic artery
  • Anterior cerebral artery
  • Middle cerebral artery
  • Posterior communicating artery
  • Anterior choroidal artery

Question.6

True about levator Ani except

AIIMS PG May-2019
A. Converge downwards & medially
B. Supports viscera
C. Made up of iliococcygeus & pubococcygeus
D. Attached to pelvic brim
Correct Ans: D
Explanation

Ans. d. Attached to pelvic brim

The levator ani is made up of three parts:

  • Iliococcygeus muscle
  • Pubococcygeus muscle
  • Puborectalis muscle

ORIGIN & Insertion:

  • The levator ani arises, in front, from the posterior surface of the superior pubic ramus lateral to the symphysis; behind, from the inner surface of the spine of the ischium; and between these two points, from the obturator fascia.
  • The fibers pass downward and backward to the middle line of the floor of the pelvis; the most posterior are inserted into the side of the last two segments of the coccyx; those placed more anteriorly unite with the muscle of the opposite side, in a median fibrous ridge called the anococcygeal body or raphe, which extends between the coccyx and the margin of the anus.

Action:Supports the viscera in pelvic cavity


Question.7

Couniad classified liver into 8 segments,which differentiates the segments anatomically

AIIMS PG May-2019
A. Portal vein
B. Hepatic vein
C. Bile duct
D. Hepatic artery
Correct Ans: B
Explanation

Ans. B) Hepatic vein

  • The Couinaud classification divides the liver into 8 functional segments.
  • The hepatic veins are found at the periphery of each segment, whereas the center has branches of the portal veins, hepatic arteries, and bile ducts.
  • The middle hepatic vein divides the liver into left and right lobes. The left hepatic vein divides the left lobe into lateral (2, 3) and medial (4a, 4b) segments. The right hepatic vein divides the right lobe into anterior (5, 8) and posterior (6, 7) segments. The caudate lobe (1) has hepatic veins that often drain directly into the IVC.


Question.8

What is the action of muscle attached at the point marked in the below image?

AIIMS PG May-2019
A. Flexion
B. Extension
C. Adduction
D. Abduction
Correct Ans: D
Explanation

Ans. d) Abduction

  • Attachments of Gluteus Medius: Origin & Insertion. Outer surface of ilium, between the posterior and anterior gluteal lines. Lateral and superior surfaces of the greater trochanter of femur
  • The function of the gluteus medius muscle is to work with other muscles on the side of the hip to help pull the thigh out to the side in a motion called hip abduction.

Question.9

Assertion – Distal pole of scaphoid mc goes to avn after scaphoid fracture

Reason-  Blood supply of scaphoid is from distal to proximal

AIIMS PG May-2019
A. Assertion is true but reason is false
B. Both assertion reason are true and reason explains assertion
C. Both assertion and reason ar false
D. Reason is true but assertion is false
Correct Ans: D
Explanation

Ans. D. Reason is true but assertion is false

  • The main blood supply to the scaphoid enters through the non-articular dorsal ridge at the waist of the bone and the volar tubercle at the distal aspect of the bone.
  • A dorsal branch of the radial artery accounts for 80% of the blood supply of the scaphoid.
  • A separate volar arterial branch to the scaphoid enters the tubercle and accounts for 20–30% of the scaphoid’s blood supply, mainly to the distal portion.
  • The proximal pole of the scaphoid relies entirely on intramedullary blood flow.
  • The unusual retrograde nature of the scaphoid’s blood supply renders it especially prone to non-union and proximal pole avascular necrosis

Question.10

Identify the Anatomical structures of heart coronal section

AIIMS PG May-2019
A. Papillary muscle
B. Cusps of valves
C. Left ventricle
D. Membranous IVseptum
Correct Ans: All:Papillary muscle, Cusps of valves, Left ventricle, Membranous IVseptum
Explanation

Ans. A. ii) Cusps of valves

B. i) Papillary muscle

C. iv) Membranous IVseptum

D. iii) Left ventricle


Question.11

Muscle not supplied by the nerve passing through Foramen marked in the image below?

AIIMS PG May-2019
A. Superior Rectus
B. Inferior Rectus
C. Medial Rectus
D. Superior Oblique
Correct Ans: D
Explanation

Ans. D. Superior oblique muscle

The trochlear nerve supplies only the superior oblique muscle.


Question.12

Golgi tendon organs used to detect ?

AIIMS PG May-2019
A. Dynamic
B. Static
C. Tension of muscle
D. All
Correct Ans: C
Explanation

Ans. C. Tension of muscle

The Golgi tendon organ (GTO) (also called Golgi organ, tendon organ, neurotendinous organ or neurotendinous spindle) is a proprioceptive sensory receptor organ that senses changes in muscle tension. It lies at the origins and insertion of skeletal muscle fibers into the tendons of skeletal muscle.


Question.13

False  about innervation of parotid gland:

AIIMS PG May-2019
A. Postganglionic parasympathetic fibre secretomotor
B. Preganglionic parasympathetic fibre relay in Otic ganglion
C. Preganglionic parasympathetic nerve begin in inferior petrosal nucleus
D. Sympathetic nerve are vasomotor
Correct Ans: C
Explanation

Ans. C. Preganglionic parasympathetic nerve begin in inferior petrosal nucleus

Nerve supply:

  • PARASYMPATHETIC:auriculo temporal nerve
  • SYMPATHETIC SUPPLY- plexus around the external carotid artery.
  • SENSORY NERVES: auriculotemporal nerve, except for parotid fascia & overlying skin which are innervated by Great auricular nerve (C2, C3).

Question.14

Identify the surface marking

AIIMS PG May-2019
A. Thoracic duct
B. Right bronchopulmonary segt
C. Right atrium border
D. Aorta
Correct Ans: C
Explanation

Ans. C. Right atrium border


Question.15

Arrange the following layers of epidermis from surface to deep layer

AIIMS PG May-2019
A. Corneocytes
B. Merkel cells
C. Melanocytes
D. Langerhans
Correct Ans: All:Corneocytes, Merkel cells, Melanocytes, Langerhans
Explanation

Ans.A ->D-> C -> B

The epidermis is composed of 4 or 5 layers, depending on the region of skin being considered.Those layers in descending order are:

  • Cornified layer (stratum corneum)
    • Composed of 10 to 30 layers of polyhedral, anucleated corneocytes (final step of keratinocyte differentiation), with the palms and soles having the most layers.
  • Clear/translucent layer (stratum lucidum, only in palms and soles)
    • This narrow layer is found only on the palms and soles. The epidermis of these two areas is known as “thick skin” because with this extra layer, the skin has 5 epidermal layers instead of 4
  • Granular layer (stratum granulosum)
    • Keratinocytes lose their nuclei and their cytoplasm appears granular. Lipids, contained into those keratinocytes within lamellar bodies, are released into the extracellular space through exocytosis to form a lipid barrier. Those polar lipids are then converted into non-polar lipids and arranged parallel to the cell surface.
  • Spinous layer (stratum spinosum)
    • Keratinocytes become connected through desmosomes and start produce lamellar bodies, from within the Golgi, enriched in polar lipids, glycosphingolipids, free sterols, phospholipids and catabolic enzymes.
    • Langerhans cells, immunologically active cells, are located in the middle of this layer.
  • Basal/germinal layer (stratum basale/germinativum).
    • Composed mainly of proliferating and non-proliferating keratinocytes, attached to the basement membrane by hemidesmosomes.
    • Melanocytes are present, connected to numerous keratinocytes in this and other strata through dendrites.
    • Merkel cells are also found in the stratum basale with large numbers in touch-sensitive sites such as the fingertips and lips.

Physiology

Question.1

Anion gap is normally calculated by using which of the following methods?

AIIMS PG May-2019
A. [K+] + [HCO3 + Cr]
B. [Na + [a- + HCO3-]
C. [Na+ – [HCO3 -+ Cl-]
D. [Mg] + [HCO3- + Cl-]
Correct Ans: C
Explanation

Ans. C. [Na+ – [HCO3 + Cl]

  • Formula for calculating anion gap =  Na+ -( HCO3 -+ Cl-)
  • Normal anion gap is (in mEq/L) = 10-12 mEq/L

Question.2

What is responsible for rhythm generation of inspiration?

AIIMS PG May-2019
A. Dorsal nucleus of vagus
B. Pre botzinger complex
C. ventral respiratory neurons
D. Pneumotaxic center
Correct Ans: B
Explanation

Ans. B. Pre botzinger complex

[Ref: Ganong 25Ie p656, 24Ie p658]

  • Rhythmic respiration is initiated by a small group of synaptically coupled pacemaker cells in the pre-Botzinger complex (pre-BOTC) on either side of the medulla between the nucleus ambiguus and the lateral reticular nucleus.
  • The main components of the respiratory control pattern generator responsible for automatic respiration are located in the medulla.
  • Rhythmic respiration is initiated by a small group of synaptically coupled pacemaker cells in the pre-Botzinger complex (pre-BOTC) on either side of the medulla between the nucleus ambiguus and the lateral reticular nucleus.

Question.3

In female adrenal gland secretes which hormone?

AIIMS PG May-2019
A. Progesterone
B. Testosterone
C. Estrogen
D. DHEA
Correct Ans: D
Explanation

Ans. D. DHEA

  • DHEA is an endogenous steroid hormone. This means it is naturally made by the body, and it spurs specific tissues or cells into action.
    • It is also known as androstenolone, 3ß-hydroxyandrost-5-en-17-one and 5-androsten-3ß-ol-17-one.
  • DHEA is one of the most abundant steroid hormones in the human body. It is produced by the adrenal glands, the gonads, and the brain.
    • It is normally found in the form of dehydroepiandrosterone sulfate (DHEAS).
  • The body holds DHEAS in reserve and converts it to specific hormones when needed.
    • It is important for creating estrogen and androgen sex hormones and contributes to the development of so-called androgenic effects, or masculinization.
    • These changes include the production of oilier skin, changes in body odor, and the growth of armpit and pubic hair.

Question.4

What is the sequence of clotting factors in coagulation pathway?

AIIMS PG May-2019
A. XII, XIII, X, XI
B. XIII, XII, XI, X
C. XII, XI, IX, X
D. X, XIII, XII, XI
Correct Ans: C
Explanation

Ans. C. XII, XI, IX, X


Question.5

Which of the following is not a monomeric intermediate filament?

AIIMS PG May-2019
A. Vimentin
B. Keratin
C. Desmin
D. Tubulin
Correct Ans: D
Explanation

Ans. D. Tubulin

  • The building block of a microtubule is the tubulin subunit, a heterodimer of a- and ß-tubulin.

Question.6

Which of the following does golgi tendon organ detect?

AIIMS PG May-2019
A. Muscle Tension
B. Dynamic muscle length
C. Static muscle length
D. Muscle action
Correct Ans: A
Explanation

Ans. A. Muscle Tension

(Ref.: Ganong 25/e p232; Guyton 13/e p697, 701).

  • Golgi tendon organ senses muscle tension.
  • The Golgi organ (also called Golgi tendon organ, GTO, tendon organ, neurotendinous organ or neurotendinous spindle) senses changes in muscle tension.
  • It is a proprioceptive sensory receptor organ that is at the origin and insertion of skeletal muscle fibers into the tendons of skeletal muscle.
  • It provides the sensory component of the Golgi tendon reflex.

Question.7

Which hormone act by crossing cell membrane –

AIIMS PG May-2019
A. Thyroxine
B. Insulin
C. Glucagon
D. Calcitonin
Correct Ans: A
Explanation

Ans. A. Thyroxine

  • Intracellular receptors are transcription factors that have binding sites for the hormone (ligand) and for DNA and function as ligand (hormone)-regulated transcription factors.
  • Steroid hormones and the steroid derivative vitamin D3 fulfill this requirement.
  • Thyroid hormones must be actively transported into the cell.

Question.8

Which of the following hormones is controlled by feedforward control?

AIIMS PG May-2019
A. ADH
B. Insulin
C. Corticosteroids
D. Prolactin
Correct Ans: A
Explanation

Ans. A. ADH

  • Osmolarity and volume status are the two greatest factors that affect ADH secretion.
  • However, a variety of other factors promote ADH secretion as well.
  • These include angiotensin II, pain, nausea, hypoglycemia, nicotine, opiates, and certain medications.
  • ADH secretion is also negatively affected by ethanol, alpha-adrenergic agonists, and atrial natriuretic peptide.
  • Ethanol’s inhibitory effect helps to explain the increased diuresis experienced during intoxicated states as well as increased free water loss; without appropriate ADH secretion, more water is excreted by the kidneys.

Question.9

Anatomical dead space measured by –

AIIMS PG May-2019
A. Bohler’s method
B. Xenon dilution technique
C. Spirometry
D. Single breath nitrogen test
Correct Ans: D
Explanation

Ans. D. Single breath nitrogen test

(Ref: Ganong 25/e p633, 634, 24/e p633, 634)

Anatomical dead space – Calculation:

  • By Bohr’s equation – Uses single breath nitrogen inhalation technique.

Xenon/Helium dilution technique:

  • Used to measure functional residual capacity of lung.

Spirometry:

  • Cannot measure residual or dead space volumes.

Question.10

Which are all the factors determining the GFR?

AIIMS PG May-2019
A. Afferent arteriolar resistance
B. Efferent arteriolar resistance
C. Arterial pressure
D. All of the above
Correct Ans: D
Explanation

Ans. D. All of the above

Determinants of GFR:

  • Increase glomerular capillary hydrostatic pressure increases GFR. Glomerular hydrostatic pressure is determined by the following variables : i) Arterial pressure, ii)Afferent arteriolar resistance; iii) Efferent arteriolar resistance.
  • Contraction of mesangial cells (therefore decrease in GFR) is produced by : – Endothelin, angiotensin II, vasopressin, norepinephrine, PAF, PDGF, thromboxane A2,PGF2, Leukotrienes C4 & D4, Histamine.

Question.11

Which of the following nucleus has cardio inhibitory function?

AIIMS PG May-2019
A. Nucleus Ambiguus
B. Nucleus Tractus Solitarius
C. Rostral Ventrolateral Medulla
D. Dorsal motor nucleus of vagus
Correct Ans: A
Explanation

Ans. A. Nucleus Ambiguus

  • The nucleus ambiguus in its “external formation” contains cholinergic preganglionic parasympathetic neurons for the heart.
  • These neurons are cardioinhibitory.
  • This cardioinhibitory effect is one of the means by which quick changes in blood pressure are achieved by the central nervous system (the primary means being changes in sympathetic nervous system activity, which constricts arterioles and makes the heart pump faster and harder).

Biochemistry

Question.1

If changes made in some of the sequence of amino acids in the chain of human proinsulin, which of the following changes will not make any difference in the  biochemical activity of the molecule?

AIIMS PG May-2019
A. Change in A1-A4
B. Change in B29,B30
C. Intra disulphide bonds
D. Change in A5 and A6
Correct Ans: A
Explanation

Answer-A-change in A1-A4

  • A6-A11 is intra disulphide bonds.A7-B7,A20-B19 are interchain bonds
  • So A1-A4 is the only amino acid not involved in insulin formation, so the best Answer is change in A1-A4

Question.2

intermediate of kreb cycle used in heme synthesis;

AIIMS PG May-2019
A. Succinyl-CoA
B. Alpha ketoglutarate
C. Citrate
D. Aspartate
Correct Ans: A
Explanation

Answer-(A) Succinyl-CoA

  • All the carbon and nitrogen atoms of the porphyrin molecule are provided by glycine
  • (a nonessential amino acid) and succinyl coenzyme A (an intermediate in the citric acid cycle) that condense to form ALA in a  reaction catalyzed by ALA synthase(ALAS)
  • Heme synthesis also requires a functional tricarboxylic acid cycle and an oxygen supply.
  • Heme synthesis starts in mitochondria with the condensation of succinyl-CoA with the amino acid glycine,  activated by pyridoxal phosphate.
  • ALA synthase is the rate-limiting enzyme of heme synthesis. ALA molecules enter the cytoplasm, where their union in the presence of ALA dehydratase yields porphobilinogen(PBG) and water molecules.

Question.3

Mother  to children’s transmission is a key feature of which pattern of inheritance?

AIIMS PG May-2019
A. Codominance
B. Autosomal dominant inheritance
C. Recessive inheritance
D. Mitochondrial inheritance
Correct Ans: D
Explanation

Ans-D-Mitocondrial inheritance , it  is the only non-chromosomal DNA in human cells.

  • Mitochondria! DNA, is always maternally inherited.
  • Mitochondrial and nuclear DNA are located in different places in the cell. During fertilization, the sperm and egg cell nuclei fuse to form an embryo.
  • The egg cell is very large compared to the sperm, so although the cells nuclei fuse, the rest of the cell mass in the embryo comes from the egg only.
  • Nuclear DNA is therefore co-inherited but the mitochondrial DNA, which is located outside of the nucleus, is always maternally inherited because all mitochondria in a foetus and later adult are derived from the mitochondria in the mother ‘s egg.
  • So, in diseases showing mitochondria! inheritance all children from affected mother will inherit the disease but it will not be transmitted from an affected father to his children.

Question.4

A young man was on high protein diet and raw eggs. After 3 days he developed weakness. Blood investigation revealed hypoglycemia. Hypoglycemia is due to inhibition of which of the following enzymes –

AIIMS PG May-2019
A. Glucose 6 phosphatase
B. Glycogen phosphorylase
C. Pyruvate Carboxylase
D. Glucokinase
Correct Ans: C
Explanation

Ans- C- Pyruvate Carboxylase

  • aminoacids in raw eggs are more easily accessed by the body. When aminoacids enter the bloodstream, they stimulate the hormone insulin, which is an anabolic hormone that drives them into the cells.
  • Raw egg contain avidin which is antagonist of biotin, so this person is deficient in biotin,
  • Carboxylase reaction need biotin, so answer is pyruvate carboxylase.

Question.5

Sickle cell anemia is a 

AIIMS PG May-2019
A. Gene deletion
B. Gene modification
C. Point mutation
D. Frame shift mutation
Correct Ans: C
Explanation

Answer- C- point mutation.

  • Sickle-cell anemia is caused by a point mutation in the ß-globin chain of hemoglobin, causing the hydrophilic amino acid glutamic acid to be replaced with the hydrophobic amino acid valine at the sixth position.
  • It is caused by a point mutation (Base substitution mutation) at the sixth position of the ß-globin chain leading to subsitution of a valine residue for a glutamic acid residue resulting in sickle hemoglobin (HbS).
  • Sickle cell anemia is an autosomal recessive disorder.

Question.6

What does not occur in 5′? 3’direction?

AIIMS PG May-2019
A. DNA repair
B. DNA replication
C. RNA editing
D. Transcription
Correct Ans: C
Explanation

Ans -C- RNA editing

  • In rna editing editosome can edit only in 3? 5 direction, along with primary rna transcript.
  • DNA replication goes in the 5′ to 3direction because DNA polymerase acts on the 3′-OH of the existing strand for adding free nucleotides.
  • In transcription the complementary RNA is created in the opposite direction, in the 5′ ? 3′ direction, matching the sequence of the sense strand with the exception of switching uracil for thymine.
  • When the strand containing the mis-match is identified, an endonuclease nicks the strand and the mis-matched nucleotide(s) is/are removed by an exonuclease.

Question.7

Hepcidin decreases iron absorption by inhibition of –

AIIMS PG May-2019
A. Hephaestin
B. Ferroportin
C. Divalent metal ion transporter
D. Transferrin
Correct Ans: B
Explanation

Ans – B- Ferroportin

  • By inhibiting ferroportin, hepcidin prevents enterocytes from allowing iron into the hepatic portal system, thereby reducing dietary iron absorption.
  • The iron release from macrophages is also reduced by ferroportin inhibition.

Question.8

Thiamine deficiency is best diagnosed by

AIIMS PG May-2019
A. Thiamine level in blood
B. Transketolase level in blood
C. Aldolase level in blood
D. Thiamine level in urine
Correct Ans: B
Explanation

Ans-B-Transketolase level in blood

  • Pyruvate dehydrogenase in carbohydrate metabolism, which catalyzes the conversion of pyruvate to acetyl CoA.
  • a-Ketoglutarate dehydrogenase in citric acid cycle, which catalyzes the conversion of a-ketogluta rate to succinyl CoA.
  • Branched-chain keto acid dehydrogenase which catalyzes the oxidative decarboxylation of branched chain keto amino acids, i.e. leucine, isoleucine and valine.
  • Thiamine diphosphate is also the coenzyme for transketolase, in the pentose phosphate pathway. There­fore, thiamine nutritional status is best assessed by erythrocyte (preferred) or whole blood transketolase activity.

Question.9

Kcat/km is  a measure  of  –

AIIMS PG May-2019
A. Enzyme efficiency
B. Speed of enzymatic reaction
C. Concentration of substrate
D. Enzyme turn over
Correct Ans: A
Explanation

Answer-A-Enzyme  efficiency

  • The  Km of  an  enzyme  is  the  concentration  of  the  substrate  that  enables  the  enzyme  to
  • Function at  half maximum  activity and is therefore  a measure  of the  specificity  of a substrate for the  enzyme” .
  • Actually  enzyme  specificity is  not measured  by alone.
  • It  is  measured  by the  ratio  Kcat  /Km  which  is  a second  order  rate constant  for  the  reaction between  substrate  and  free  enzyme.
  • This  ratio  is  important,  for  it  provides  a direct  measure of  enzyme  eficiency and  specificity.
  • Note:  Km, is  turnover  number  and  measures  tJre  rate  of  the  catalytic  process

Question.10

Enzyme involved in gluconeogenesis are all except:

AIIMS PG May-2019
A. Pyruvate carboxylase
B. PEP carboxykinase
C. Pyruvate kinase
D. Glucose-6-phosphatase
Correct Ans: C
Explanation

Answer-C-Pyruvate kinase

  • Mitochondrial  pyruvate  carboxylase catalyzes  the carboxylation  of  Pyruvate  to  Oxaloacetate, It  is  an  ATP-requiring  reaction, Biotin  is  the  coenzyme.
  • Phosphoenolpyruvate  Carboxykinase: Catalyzes  the  decarboxylation  and phosphorylation of oxaloacetate  to  phosphoenolpyruvate(PEPCK)  (Cytosol)  using  GTP as the  phosphate  donor.
  • The conversion of glucose-6-phosphate  to  glucose  is catalyzed  by glucose  6-phosphatase

Question.11

Which of the  following  leads  to  an  increase  in enzyme  activity  –

AIIMS PG May-2019
A. Increase in temperature
B. Decrease in activation energy
C. Extremes of pH value
D. Low substrate concentration
Correct Ans: B
Explanation

Answer-B-Decrease in activation  energy

  • The enzymes  speed  up chemical  reactions by  lowering  the  magnitude  of the  activation  energy  banier,  i.e.,free energy  of Activation.

Question.12

The letters A through E designate certain regions on the titration curve for glycine (shown below). Which one of the following statements concerning this curve is correct?

AIIMS PG May-2019
A. Represents the region where glycine is deprotonated
B. Point B represents a region of minimal buffering
C. Point C represents the region where the net charge on glycine is zero
D. Point D represents the pK of glycine’s carboxyl group
E. Point E represents the pI for glycine
Correct Ans: C, Point E represents the pI for glycine
Explanation

Answer-C-Point C represents the region where the net charge on glycine is zero

  • C represents the isoelectric  point or pI, and as such is midway between pK1
  • and pK2 for this monoamino monocarboxylic acid.
  • Glycine is fully protonated at Point A. Point B represents a region of maximum buffering, as does Point D.
  • Point E represents the region where glycine is fully deprotonated.

Question.13

Assertion : Central dogma is the flow of information from DNA to mRNA and then decoding the information present in mRNA in the form of protein.

Reason : In retroviruses, reverse of central dogma occurs.

AIIMS PG May-2019
A. If both the assertion and the reason are true and the reason is a correct explanation of the assertion
B. If both assertion and reason are true but the reason is not the correct explanation of the assertion
C. If the assertion is true but the reason is false
D. If both the assertion and reason are false
E. If the assertion is false but reason is true
Correct Ans: B, If the assertion is false but reason is true
Explanation

Answer: B. If both assertion and reason are true but the reason is not the correct explanation of the assertion

  • Biosynthesis of protein is under direct control of DNA in most cases or else under the control of genetic RNA where DNA is absent. Sequences of bases in a particular segment of a polynucleotide chain will determine the sequence of amino acids in a particular polypeptide.
  • The relationship is popularly known as central dogma. Flow of information is one way i.e., from DNA, information is transferred to RNA (mRNA) and from RNA to protein.
  •  Temin (1970) reported that retroviruses operate a central dogma reverse or teminism inside host cells. Genomic RNA of these viruses first synthesizes DNA through reverse transcription. DNA then transfers information to messenger RNA which takes part in translation of the coded information to form polypeptide.

(i) One way flow of information (central dogma)

DNA(transcription) ? RNA (translation) ? proteins

(i) Reverse flow of transcription information

DNA(transcription) ?RNA(translation) ? proteins

Microbiology

Question.1

Correct order of gram staining is –

AIIMS PG May-2019
A. Gention violet ? IodinE ? Carbol fuchsin
B. Iodine ? Gention violet ? Carbol fuchsin
C. Carbol fuchsin ? Iodine ? Gention violet
D. Carbol fuchsin ? Gention violet ? Iodine
Correct Ans: A
Explanation

Answer-A-Gention violet ? IodinE ? Carbol fuchsin

  1. Application of the primary stain (crystal violet). Gention violet also known as crystal violet stains all cells blue/purple
  2. Application of mordant: The iodine solution (mordant) is added to form a crystal violet-iodine (CV-I) complex; all cells continue to appear blue.
  3. Decolorization step: The decolorization step distinguishes gram-positive from gram-negative cells. The organic solvent such as acetone or ethanol, extracts the blue dye complex from the lipid-rich, thin-walled gram-negative bacteria to a greater degree than from the lipid-poor, thick-walled, gram-positive bacteria.  The gram-negative bacteria appear colorless and gram-positive bacteria remain blue.
  4. Application of counterstain (safranin): The red dye safranin stains the decolorized gram-negative cells red/pink; the gram-positive bacteria remain blue.

NOTE ? If you are struggling to remember the staining reagents used in this procedure and their order you can remember this sentence “Come In And Stain” i.e. the order is Crystal violet, Iodine, Alcohol/Acetone and the final one is Safranin.


Question.2

Antibody in chronic allergy ?

AIIMS PG May-2019
A. IgM
B. IgA
C. IgG
D. IgE
Correct Ans: D
Explanation

Answer-D-IgE

  • When someone has allergies, their immune system makes an antibody called immunoglobulin E (IgE). These antibodies respond to allergens. The symptoms that result are an allergic reaction.
  • IgE also has an essential role in type I hypersensitivity, which manifests in various allergic diseases, such as allergic asthma, most types of sinusitis, allergic rhinitis, food allergies, and specific types of chronic urticaria and atopic dermatitis.

Question.3

Identify the given image and transmission associated with image?

AIIMS PG May-2019
A. Malaria
B. Filariasis
C. Dengue
D. A And C
Correct Ans: B
Explanation

Answer- B-i.e., Filariasis , the given image is of culex mosquito.

Mosquito species Disease transmitted
Anopheles Malaria, filariasis (not in India), arboviruses of febrile and encephalitic disease
Culex Bancroftian filariasis, Japanese encephalitis, West-Nile fever, St.
Louis encephalitis, western equine encephalitis
Aedes yellow fever (in Africa), Dengue, Dengue haemorrhagic fever, Chickengunya, Chickengunya haemorrhagic fever, Rift valley fever, Sindbis, Bancroftian filariasis (not in India)
Mansonia Brugian filariasis, Bancroftian filariasis, Chikungunya

Question.4

Chancre redux is a clinical feature of –

AIIMS PG May-2019
A. Early relapsing syphilis
B. Late syphilis
C. Chancroid
D. Recurrent herpes simplex infection
Correct Ans: A
Explanation

Ans. is ‘a’ i.e., Early relapsing syphilis

  • Chancre redux is the appearance of relapsing lesion at the site of the healed lesion, it is due to relapse of original infection (not due to reinfection)

Question.5

Infectious cause of erythema multiforme in given image is –

AIIMS PG May-2019
A. Staphylococcus
B. TB
C. HSV
D. EBV
Correct Ans: C
Explanation

Answer-C-Herpes simplex virus

  • Herpes simplex is the primary cause of erythema multiforme, and the virus is present in 70 percent of recurrent erythema multiforme cases. Both types of herpes simplex virus (HSV) can cause the condition, but HSV-1, which also causes cold sores, is responsible for most cases.

Causes of Erythema multiforme

  • Idiopathic ? Most common cause
  • Viral ? HSV (most important) HBV, Mumps, Adenovirus
  • Bacteria ? Streptococci, tuberculosis
  • Fungal ? Coccidioidomycosis, Histoplasmosis.
  • Drugs ? Antibiotics (Sulphonamide), Phenytoin, NSAIDS.
  • Autoimmune disease ? SLE, thyroiditis, RA
  • Others ? Sarcoidosis, Pregnancy, Malignancy.

Question.6

Clostridium difficile diarrhoea associated with:

AIIMS PG May-2019
A. Aminopenicillins
B. Carbapenems
C. Macrolide
D. Fluoroquinolones
Correct Ans: D
Explanation

Answer-D-fluoroquinolones,

  • Clostridium difficile colitis results from a disturbance of the normal bacterial flora of the colon, colonization by C difficile, and the release of toxins that cause mucosal inflammation and damage.
  • It is a spore-forming bacillus that is responsible for the development of antibiotic-associated diarrhea and colitis.
  • The antibiotics that most often lead to C. difficile infections include fluoroquinolones, cephalosporins,penicillins and clindamycin. Once established, C. difficile can produce toxins that attack the lining of the intestine

Question.7

The dimorphic fungus exists in two phases: a unicellular yeast form at 37 C and a mycelium at 25 C; the fungus is:

AIIMS PG May-2019
A. Malassezia furfur
B. Cryptococcus neoformans
C. Aspergillus
D. Histoplasma capsulatum
Correct Ans: D
Explanation

Answer-D- Histoplasma capsulatum

  • Dimorphic fungi are fungi that can exist in the form of both mold and yeast. An example is Penicillium marneffei, a human pathogen that grows as a mold at room temperature, and as a yeast at human body temperature.

Mnemonics: Body Heat Probably (Changes) Shape

  • Blastomyces dermatitidis, Histoplasma capsulatum, Paracoccidioides brasiliensis, (Coccidioides immitis) is in parentheses because it changes to a spherule of endospores, not yeast, in the heat), Sporothrix schenckii.

Question.8

Identify the disease caused by the given organism?

AIIMS PG May-2019
A. Giardia
B. Leishmania
C. Trypanosoma
D. Plasmodium
Correct Ans: B
Explanation

Answer-B-Leishmania

  • It is a disease caused by protozoan parasites of the genus Leishmania and spread by the bite of certain types of sandflies.
  • The disease can present in three main ways: cutaneous, mucocutaneous, or visceral leishmaniasis(Kala azar/Black fever).
  • The cutaneous form presents with skin ulcers, while the mucocutaneous form presents with ulcers of the skin, mouth, and nose, and the visceral form starts with skin ulcers and then later presents with fever, low red blood cells, and enlarged spleen and liver.
  • Microscopic examination:direct visualization of the intracellular Amastigotes (Leishman-Donovan bodies).Amastigotes( formed after the macrophage phagocytizes an infective promastigote) are seen within blood and spleen monocytes or, less commonly, in circulating neutrophils and in aspirated tissue macrophages. They are small, round bodies 2–4 µm in diameter with indistinct cytoplasm, a nucleus, and a small, rod-shaped kinetoplast.
  • Culture:Growth of Promastigotes in days to weeks.Promastigotes are characterized by a flagellum and a kinetoplast anterior to the nucleus. They are the infective stage to humans.

Question.9

Typical eschar in given image is of:

 

AIIMS PG May-2019
A. louse-borne typhus
B. Scrub typhus
C. murine) typhus
D. None of these
Correct Ans: B
Explanation

Answer-B-scrub typhus

  • It is a form of typhus caused by Orientia tsutsugamushi first isolated and identified in 1930 in Japan
  • Scrub typhus is transmitted by some species of trombiculid mites (“chiggers”, particularly Leptotrombidium deliense), which are found in areas of heavy scrub vegetation.
  • The bite of this mite leaves a characteristic black eschar that is useful for making the diagnosis.
  • Scrub typhus is endemic to a part of the world known as the “tsutsugamushi triangle” (after the name “Orientia tsutsugamushi” (formerly “Rickettsia tsutsugamushi”), the obligate intracellular gram-negative bacterium causing same), which extends from northern Japan and far-eastern Russia in the north, to the territories around the Solomon Sea into northern Australia in the south, and to Pakistan and Afghanistan in the west.

Question.10

Mw vaccine is made from which bacteria?

AIIMS PG May-2019
A. M. Welchii
B. M. Bovis
C. M. Indicus pranii
D. None of these
Correct Ans: C
Explanation

Answer-C-M. Indicus pranii

  • Killed Mycobacterium indicus pranii (previously known as Mycobacterium w, popularly known as Mw) vaccine has earlier been investigated in genital warts with encouraging results
  • Mycobacterium w vaccine, a useful adjuvant to multidrug therapy in multibacillary leprosy.

Question.11

All are seen in sickle cell anemia except ?

AIIMS PG May-2019
A. Target cells
B. Malaria
C. Reticulocytosis
D. Jaundice
Correct Ans: B
Explanation

Answer-B-malaria

  • The possession of the sickle cell allele protects these individuals against malaria because the parasite dies when potassium leaks out of RBCs as they become sickle shaped.
  • Types,of anemia that have protective effect against P. falciparum malaria : – G6PD deficiency, Sickle cell anemia, Thalassemia, HbC, Pyruvate kinase deficiency

Question.12

Cerebral malaria is caused by?

AIIMS PG May-2019
A. Plasmodium falciparum
B. Plasmodium ovale
C. Plasmodium vivax
D. Plasmodium malariae
Correct Ans: A
Explanation

Answer- A-Plasmodium falciparum

  • Plasmodium falciparum REF: Harrisons 17th edition, chapter 203
  • Cerebral Malaria: Coma is a characteristic and ominous feature of falciparum malaria and, despite treatment, is associated with death rates of 20% among adults and 15% among children. Any obtundation, delirium, or abnormal behavior should be taken very seriously. The onset may be gradual or sudden following a convulsion

Pharmacology

Question.1

Preferred drug for the treatment of uncomplicated grade 2 hypertension in a 48 year old man is-

AIIMS PG May-2019
A. Chlorthalidone
B. Triamterene
C. Spironolactone
D. Furosemide
Correct Ans: A
Explanation

Ans. A. Chlorthalidone 

  • Thiazide diuretics are inexpensive, are generally well tolerated, and are recommended as a first-line therapy in the treatment of hypertension in the elderly population.
  • Thiazide diuretics are as effective as any drug for first-line treatment of hypertension in the elderly population is the Antihypertensive and Lipid-Lowering Treatment to Prevent Heart Attack Trial (ALLHAT) study.
  • Patients who received the diuretic had a lower incidence of CV events (secondary outcomes) compared with the other groups. The diuretic treatment group had lower HF rates compared with the CCB group

Question.2

Propranolol is drug of choice for-

AIIMS PG May-2019
A. Ulcerated infantile hemangioma
B. Lymphangioma
C. Pyogenic granuloma
D. Capillary malformation
Correct Ans: A
Explanation

Ans. A. Ulcerated infantile hemangioma

[Ref: https://www.ncbi.nlm.nih.gov/pmc/articles/PMC3800297/]

  • Infantile hemangiomas (IHs) are the most common benign pediatric soft-tissue tumors.
  • Ulceration—the most frequent complication of IH—tends to heal poorly and is associated with pain, bleeding, infection, and scarring.
  • Mainstay treatment modalities include propranolol (ß-blocker) and corticosteroids, whose effectiveness is countered by a need for long-term medication and risk of systemic adverse effects and ulcer recurrence.

Question.3

All are used for postpartum hemorrhage except.

AIIMS PG May-2019
A. Misoprostol
B. Dinoprostone
C. Prostaglandin F2 alpha
D. Oxytocin
Correct Ans: B
Explanation

Ans. B. Dinoprostone

  • Postpartum hemorrhage, the loss of more than 500 mL of blood after delivery, occurs in up to 18 percent of births and is the most common maternal morbidity in developed countries.

Dinoprostone:

  • Naturally occurring prostaglandin E2 (PGE2). It has important effects in labour.
  • It also stimulates osteoblasts to release factors which stimulates bone resorption by osteoclasts.
  • As a prescription drug it is used as a vaginal suppository, to prepare the cervix for labour and to induce labour.

Indication:

  • For the termination of pregnancy during the second trimester (from the 12th through the 20th gestational week as calculated from the first day of the last normal menstrual period), as well as for evacuation of the uterine contents in the management of missed abortion or intrauterine fetal death up to 28 weeks of gestational age as calculated from the first day of the last normal menstrual period.
  • In the management of nonmetastatic gestational trophoblastic disease (benign hydatidiform mole).
  • Other indications include improving the cervical inducibility (cervical “ripening”) in pregnant women at or near term with a medical or obstetrical need for labor induction, and the management of postpartum hemorrhage.

Question.4

Most commonly implicated drug for acute liver failure is-

AIIMS PG May-2019
A. Paracetamol
B. Valproate
C. Warfarin
D. Tetracyclines
Correct Ans: A
Explanation

Ans. A. Paracetamol 

  • Acute liver failure after administration of paracetamol at the maximum recommended daily dose in adults.
  • Paracetamol is the most commonly used analgesic and antipyretic in the world; it can be bought without prescription in most countries despite being the commonest cause of acute liver failure in western Europe.
  • Prescribing information suggests that it is safe to use in adults in divided doses that total 4 g daily.
  • Malnutrition, starvation, chronic alcohol misuse, and concomitant use of drugs that induce cytochrome P450 enzymes increase the risk of hepatotoxicity induced by paracetamol.

Question.5

There was an outbreak of MRSA in the hospital and it was found that a nurse of NICU had MRSA colonisation of anterior nares. What is the best treatment?

AIIMS PG May-2019
A. Topical bacitracin
B. Oral Vancomycin
C. Inhaled colistin
D. IV cefazolin
Correct Ans: A
Explanation

Ans. A. Topical bacitracin

  • MRSA (methicillin-resistant Staphylococcus aureus) infection is a life-threatening bacterial infection caused by Staphylococcus aureus that is resistant to the antibiotic methicillin. Severe cases usually require hospitalization for treatment.
  • MRSA infections are usually treated with systemic antibiotics, either orally or intravenously.
  • Topical bacitracin in combination with neomycin and polymyxin B might be added to the therapy.

Question.6

Fixed drug eruptions are frequently seen with?

AIIMS PG May-2019
A. Penicillin
B. Sulfonamide
C. Cetirizine
D. Roxithromycin
Correct Ans: B
Explanation

Ans. B. Sulfonamide

Drugs causing fixed drug eruption

  • Paracetamol (Phenacetin)
  • Sulfonamides
  • NSAIDs
  • Aspirin
  • Barbiturates
  • Dapsone
  • Tetracyclines
  • Phenylbutazone

Question.7

All are topical hemostatic agents except?

AIIMS PG May-2019
A. Bone wax & patty
B. HemCon bandage
C. Quikclot
D. Fixclot
Correct Ans: D
Explanation

Ans. D. Fixclot 

  • Topical hemostatic agents are used when surgical hemostasis is inadequate or impractical. The majority of routine, elective operations are performed in patients with normal hemostasis and with minimal blood loss.
  • The two main categories of topical hemostatic agents are physical agents, which promote hemostasis using a passive substrate, and biologically active agents, which enhance coagulation at the bleeding site.

Question.8

In iron poisoning drug Desferoxamine is given.what is its mechanism of action-

AIIMS PG May-2019
A. By binding with trivalent Fe3+
B. -by inhibiting Hepcidin
C. -by inhibiting DMT1
D. -by inhibiting ferroportin
Correct Ans: A
Explanation

Ans. A. By binding with trivalent Fe3+

  • Deferoxamine works in treating iron toxicity by binding trivalent (ferric) iron (for which it has a strong affinity), forming ferrioxamine, a stable complex which is eliminated via the kidneys.
  • 100 mg of deferoxamine is capable of binding approximately 8.5 mg of trivalent (ferric) iron.
  • Deferoxamine works in treating aluminum toxicity by binding to tissue-bound aluminum to form aluminoxamine, a stable, water-soluble complex.
    • The formation of aluminoxamine increases blood concentrations of aluminum, resulting in an increased concentration gradient between the blood and dialysate, boosting the removal of aluminum during dialysis.
  • 100 mg of deferoxamine is capable of binding approximately 4.1 mg of aluminum.

Question.9

A bank employee felt depressed with no interest in activities came to AIIMS OPD. He was started on Escitalopram. Which of these adverse effects cannot be explained with escitalopram?

AIIMS PG May-2019
A. Vivid dreaming
B. Anorgasmia
C. Sialorrhea
D. Nausea
Correct Ans: C
Explanation

Ans. C. Sialorrhea

  • Escitalopram belongs to a class of drugs known as selective serotonin reuptake inhibitors (SSRI).
  • Improves your energy levels and feelings of well-being and decrease nervousness.
  • Used to treat depression and anxiety.

MOA:

  • Works by helping to restore the serotonin balance in the brain.

S/E:

  • Headache, Nausea, Ejaculation disorder, Somnolence, Insomnia, Dry mouth, Constipation, Fatigue, Libido decreased, Inability to achieve orgasm, Gas (flatulence), Toothache, Weight gain, Menstrual disorder, Neck/shoulder pain, Runny nose & Flu-like syndrome.

Question.10

Chlorpromazine act on which of the following receptors-

  1. D2 and 5HT2 receptors
  2. GABA and Beta-adrenergic receptors
  3. Muscarinic M1 and alpha-adrenergic receptors
  4. H1 receptors
AIIMS PG May-2019
A. 1,3 and 4 are correct
B. Only 2 is correct
C. 1 and 2 are correct
D. All are correct
Correct Ans: A
Explanation

Ans. A. 1, 3 and 4 are correct

Chlorpromazine:

  • Chlorpromazine is a psychotropic agent indicated for the treatment of schizophrenia. It also exerts sedative and antiemetic activity.
  • Chlorpromazine has actions at all levels of the central nervous system-primarily at subcortical levels-as well as on multiple organ systems.
  • Chlorpromazine has strong antiadrenergic and weaker peripheral anticholinergic activity; ganglionic blocking action is relatively slight.
  • It also possesses slight antihistaminic and antiserotonin activity.
  • Traditional antipsychotic agent with anti-emetic activity. 

MOA:

  • Exerts its antipsychotic effect by blocking postsynaptic dopamine receptors in cortical and limbic areas of the brain, thereby preventing the excess of dopamine in the brain.
  • This leads to a reduction in psychotic symptoms, such as hallucinations and delusions.
  • Chlorpromazine appears to exert its anti-emetic activity by blocking dopamine receptors in the chemical trigger zone (CTZ) in the brain, thereby relieving nausea and vomiting.

Question.11

Which of the following is/are adverse effect of SGLT2 inhibitors?

  1. Ketoacidosis
  2. Urosepsis
  3. Fournier’s gangrene
  4. Fournier’s gangrene
AIIMS PG May-2019
A. 1,2 and 4 are correct
B. 1, 2 and 3 are correct
C. Only 4 is correct
D. All are correct
Correct Ans: B
Explanation

Ans. B. 1, 2 and 3 are correct

Sodium glucose co-transporter-2:

  • Sodium  glucose co-transporter-2 [SGLT-2] present in proximal tubules.
  • Glucose is freely filtered across glomerulus & is 100% reabsorbed in proximal tubules, via SGLT-2.

Sodium glucose co-transporter-2 inhibitors (SGLT-2 inhibitors):

  • SGLT-2 inhibitors acts by inhibiting this transporter ? Causing glucosuria in diabetics ? causes weight loss.
  • Effective orally.
  • Efficacy reduced in renal failure.

Side effects:

  • Ketoacidosis, Urosepsi, Fournier’s gangrene, Increased incidence of UTI & genital infections.

Question.12

Assertion: Large doses of acyclovir are recommended for treating genital herpes in HIV positive patients.

Reason: Frequent recurrence of genital herpes is seen in HIV patients

 

AIIMS PG May-2019
A. Both Assertion and Reason are true and Reason is correct explanation of Assertion
B. Assertion is true & reason is not the right explanation for assertion
C. Assertion is false & reason is also false
D. Both assertion & reason are false
Correct Ans: A
Explanation

Ans. A. Both Assertion and Reason are true and Reason is correct explanation of Assertion

ACYCLOVIR

  • Antiviral drug.
  • Requires virus-specific enzyme for conversion to active metabolite inhibiting DNA synthesis & viral replication.
  • Varicella-zoster viruses are not as sensitive as herpes simplex viruses.

INDICATIONS:

Herpes simplex:

  • For genital primary infections.
  • Oral acyclovir – Faster healing rate than acyclovir ointment.

For herpes simplex keratitis:

  • Local/oral therapy successful.
  • Herpes simplex encephalitis & herpes simplex neonatorum:
  • Must be treated intravenously.

Frequently relapsing herpes simplex (labial/genital):

  • Long-term, prophylactic acyclovir administration.

Question.13

Mention the true/false statements about digoxin toxicity?

AIIMS PG May-2019
A. Earliest manifestation of digoxin toxicity are gastrointestinal symptoms
B. Non specific vision changes may be noted in digoxin toxicity
C. Early toxicity may not correlate with serum levels
D. Neurological symptoms may occur without corresponding cardiovascular changes
Correct Ans: All:Earliest manifestation of digoxin toxicity are gastrointestinal symptoms, Non specific vision changes may be noted in digoxin toxicity, Early toxicity may not correlate with serum levels, Neurological symptoms may occur without corresponding cardiovascular changes
Explanation

Ans. All are true about digoxin toxicity.

DIGOXIN TOXICITY:

Features:

  • Generally unwell & lethargy.
  • Nausea & vomiting.
  • Confusion.
  • Yellow-green vision.
  • Arrhythmias (e.g. AV block, bradycardia)
  • Dizziness.

Precipitating factors:

  • Renal disease   
  • Hypokalaemia
  • Hypomagnesemia     
  • Hypoalbuminemia
  • Hypothermia
  • Hypothyroidism
  • Hypercalcemia.
  • Hypernatremia
  • Acidosis.
  • Myocardial ischaemia.
  • Partial AV block.

Drugs:

  • Amiodarone.
  • Quinidine.
  • Verapamil.
  • Spironolactone.
  • Furosemide.
  • Hydrochlorothiazide  – Compete with DCT secretion, hence reducing excretion.

Management

  • Digibind.
  • Correct ventricular arrhythmia by lignocaine.
  • Bradyarrhythmias by propanolol.
  • Atrial tachyarrhythmias by atropine.
  • Phenytoin.
  • Monitor K+

Question.14

Mention the true/false statements about drug resistant malaria?

AIIMS PG May-2019
A. Quinine with clindamycin or doxycycline is still effective treatment
B. Artemether with lumefantrine is useful
C. Monotherapy with artemisinin derivatives is not useful due to high relapse rate
D. All of the above
Correct Ans: D
Explanation

Ans:  A, B & C are True

A & C are false.

Multidrug resistance of P. Falciparum is seen when the parasite is resistant to more than two operational antimalarial compounds of different chemical classes and modes of action.


Question.15

Match the following drugs with their ocular adverse effects-

Drug Adverse Effect
1. Amiodarone A. Blepharoconjunctivitis
2. Hydroxychloroquine B. Angle closure glaucoma
3. Systemic steroids C. Retinopathy
4. Digoxin D. Optic neuritis
E. Yellow vision E. Yellow vision
F. Cataract F. Cataract
G. Corneal microdeposits G. Corneal microdeposits

 

AIIMS PG May-2019
A. 1-B, 2-D, 3-A, 4-C
B. 1-C, 2-F, 3-A, 4-E
C. 1-G, 2-C, 3-F, 4-E
D. 1-F, 2-G, 3-B, 4-C
Correct Ans: C
Explanation

Ans. C. 1-G, 2-C, 3-F, 4-E

  • Amiodarone – Corneal microdeposits.
  • Hydroxychloroquine – Retinopathy
  • Systemic steroids – Cataract
  • Digoxin – Yellow vision.

Question.16

Aspirin and phenobarbitone are acidic drugs whereas diazepam is a basic drug. Mention the true/false statements about these drugs?

AIIMS PG May-2019
A. Aspirin is present in mainly non-ionised form in stomach, hence can be easily absorbed
B. Diazepam is mostly absorbed from intestine
C. Phenobarbitone can be absorbed from the stomach but most of the absorption occurs in small intestine due to its large surface area
D. All of the above
Correct Ans: D
Explanation

Ans: A, B & C are True

D & E are false

  • Acidic drugs with pKa 3.5:
    • Largely unionized at acid gastric pH & absorbed from stomach.
    • Eg: Aspirin.
  • Basic drugs with pKa 10:
    • Largely ionized & are absorbed only in intestines.
    • Eg: Atropine
  • Unionized form is lipid soluble & ionized form is water soluble.
    • Medium is same, hence drugs cross membrane.

Acidic drugs are lipid soluble (un-ionized form) in acidic medium.

  • Eg: In acidic gastric pH these acidic drugs are more likely to be absorbed from stomach.
    • Due to presence of unionized (lipid soluble) form.
  • Hence, aspirin is more likely to be absorbed in stomach than morphine or atropine (basic drugs).
    • This is reason for aspirin-induced gastric mucosal irritation. 

Ion trapping:

  • Unionized drugs reverts back to ionized form within cell (pH 7.0) before passing to ECF.
    • Referred as “ion trapping”.
    • Ie., Weak electrolyte crossing a membrane to encounter a pH from which it is not able to escape easily.
  • Basic drugs attain higher concentration intracellularly (pH 7.0 vs 7.4 of plasma).

Question.17

Match the following drugs with organism they are used for-

Drug Organism
1. Praziquantel A. Filaria
2. Diethylcarbamazine B. Giardia
3. Nitazoxanide C. Strongyloides
4. Mebendazole D. Tapeworms
E. Leishmania
F. Ascaris lumbricoides
AIIMS PG May-2019
A. 1-B, 2-D, 3-A, 4-C
B. 1-D, 2-A, 3-B, 4-F
C. 1-G, 2-C, 3-F, 4-E
D. 1-F, 2-G, 3-B, 4-C
Correct Ans: B
Explanation

Ans. B. 1-D, 2-A, 3-B, 4-F

  • Praziquantel – Tapeworms
  • Diethylcarbamazine – Filaria
  • Nitazoxanide – Giardia
  • Mebendazole – Ascaris lumbricoides

Question.18

Match the following anaesthetic agents with its appropriate property?

Drug Adverse Effect
1. Propofol A. Rigid chest syndrome
2. Fentanyl B. Pulmonary vasoconstriction
3. Midazolam C. Avoided in patients with egg Allergy
4. Nitrous oxide D. Hypotension
E. Dissociative anaesthesia
  F. Adrenal insufficiency

 

AIIMS PG May-2019
A. 1-C, 2-A, 3-D, 4-B
B. 1-D, 2-A, 3-B, 4-F
C. 1-G, 2-C, 3-F, 4-E
D. 1-F, 2-G, 3-B, 4-C
Correct Ans: A
Explanation

Ans. A. 1-C, 2-A, 3-D, 4-B 

  • Propofol – Avoided in patients with egg Allergy
  • Fentanyl – Rigid chest syndrome
  • Midazolam – Hypotension
  • Nitrous oxide – Pulmonary vasoconstriction

Question.19

Match the following drug poisonings with their antidotes-

Drug overdose Antidote
1. HCN A. Trimethadione
2. Paracetamol B. Nalorphine
3. Morphine C. Bupropion
4. Nicotine D. Diazepam
E. N-acetylcysteine
  F. Amyl nitrite

 

AIIMS PG May-2019
A. 1-B, 2-D, 3-A, 4-C
B. 1-D, 2-A, 3-B, 4-F
C. 1-G, 2-C, 3-F, 4-E
D. 1-F, 2-E, 3-B, 4-A
Correct Ans: D
Explanation

Ans. D. 1-F, 2-E, 3-B, 4-A

  • HCN – Amyl nitrite
  • Paracetamol  – N-acetylcysteine
  • Morphine  – Nalorphine
  • Nicotine – Trimethadione

Forensic Medicine

Question.1

Identify the given image of post mortem staining-

 

 

AIIMS PG May-2019
A. Pallor mortis
B. Algor mortis
C. Postmortem hypostasis
D. Postmortem marbling
Correct Ans: C
Explanation

Answer- C’- hypostasia-(suggligation)

  • The accumulation of fluid or blood in the lower parts of the body or organs under the influence of gravity, as occurs in cases of poor circulation or after death.
  • hypostasis or suggillation, is the fourth stage of death and one of the signs of death. It is a settling of the blood in the lower, or dependent, portion of the body postmortem, causing a purplish red discoloration of the skin.
  • When the heart stops functioning and is no longer agitating the blood, heavy red blood cells sink through the serum by action of gravity. The blood travels faster in warmer conditions and slower in colder conditions.

Question.2

In the sequence in following clauses, which of the following is incorrect:

AIIMS PG May-2019
A. Second ? Permanent privation of the sight of either eye
B. First ? Emasculation
C. Eight ? Fracture or dislocation of a bone or tooth
D. Sixth ? Permanent disfiguration of the head or face
Correct Ans: C
Explanation

Answer- C-Fracture or dislocation of a bone or tooth is the seventh clause of grievous hurt 

Sec-320 ? following kinds of hurt only are designated as “grievous”:-

  1. Firstly.-Emasculation
  2. Secondly.-Permanent privation of the sight of either eye.
  3. Thirdly.-Permanent privation of the hearing of either ear.
  4. Fourthly.-Privation of any member or joint.
  5. Fifthly.-Destruction or permanent impairing of the powers of any member or joint.
  6. Sixthly.-Permanent disfiguration of the head or face.
  7. Seventhly.-Fracture or dislocation of a bone or tooth.
  8. Eighthly.-Any hurt which endangers life or which causes the sufferer to be during the space of twenty days in severe bodily pain, or unable to follow his ordinary pursuits.

Question.3

Paradoxical undressing seen in-

AIIMS PG May-2019
A. Immersion syndrome
B. Heat cramps
C. Hypothermia
D. Heat exhaustion
Correct Ans: C
Explanation

Ans- C-hypothermia

  • Paradoxical undressing is a term for a phenomenon frequently seen in cases of lethal hypothermia. Shortly before death, the person will remove all their clothes, as if they were burning up, when in fact they are freezing.

Question.4

What is the most common cause of parasuicide?

AIIMS PG May-2019
A. Drug ingestion
B. Hanging
C. Cutting wrist
D. Firearms
Correct Ans: A
Explanation

Ans. A. Drug ingestion

  • Parasuicide is a suicide attempt or gesture and self-harm where there is no result in death. It is a non7fatal act in which a person deliberately causes injury to him or herself or ingests any prescribed or generally recognized therapeutic dose in excess.
  • It is considered to be a serious public health issue. Parasuicide is the strongest known indicator for a future successful suicide attempt.
  • Examples of suicidal gestures include cutting, where the cut is not deep enough to cause significant blood loss, or taking a non-lethal overdose of medication.’- http://en.wikipedia.org/wiki/Parasuicide
  • `Maximum cases of near-suicides are due to drug overdose. In India, wrist cutting is equally common, but drug overdose is by far the most common cause of parasuicide in the world.’- Methods used for parasuicide: results of the WHO/EURO Multicentre Study on Parasuicide.

Question.5

Max damage  is  done  by  which Poison?

AIIMS PG May-2019
A. Irritant poison
B. Corrosive poison
C. Alcohol
D. Opioid
Correct Ans: B
Explanation

Ans.  is’B’i.e.,Corrosive poison

  • Corrosives  fix,  destroy  and erode  the  surface  with  which  they  come  in contact.  Thus, they  cause  extensive  tissue damage.
  • Irritants  produce  less  severe  damage  than  corrosives  and  produces ‘symPtoms simulating  gastroenteritis’

Question.6

In post mortem body traumatic injury of face with one eye missing and some injury on mouth and nose ( right eye ball was missing , blood in socket area ) Cause of injury-

AIIMS PG May-2019
A. Blunt rupture to eyeball
B. Evisceration by sharp weapon
C. Post mortem artefact
D. None of the Above
Correct Ans: B
Explanation

Answer B. Evisceration by sharp weapon

  • In blunt rupture of eye the complete eye will not be missing. Also since the blood is present in the socket so post mortem artifact is ruled out.
  • Evisceration by Sharp object  the eye ball will be completely missing. Some injury to mouth or nose can be due to trauma in those areas.

Pathology

Question.1

Best test that shows the integrity of intrinsic pathway of clotting mechanism-

AIIMS PG May-2019
A. bleeding time
B. aPTT
C. prothrombin time
D. clotting time
Correct Ans: C
Explanation

Answer-C- prothrombin time

The PT measures the time necessary to generate fibrin after activation of factor VII. It measures the integrity of the “extrinsic” and  “common” pathways (factors VII, V, X, prothrombin, and fibrinogen).


Question.2

Patch test image is given and asked when will be readings taken-

AIIMS PG May-2019
A. 24 hrs and 48hrs
B. 48 hrs and 96hrs
C. 48hrs and 72 hrs
D. 48hr & 96hrs
Correct Ans: A
Explanation

Answer-A-24 hrs and 48hrs

There are 3 appointments,  marks must be visible at the third appointment, usually 24–48 hours later (72–96 hours after application).


Question.3

Vascular event of inflammation in order-

Immediate transient, delayed prolonged, immediate prolonged ,delayed prolonged

 

AIIMS PG May-2019
A. mast cell activation lead to release of histamine bradykinin
B. leukocytes mediated injury
C. sun exposure /heat stroke lead to mild endothelial injury
D. direct endothelial injury by bacterial toxin
Correct Ans: All:mast cell activation lead to release of histamine bradykinin, leukocytes mediated injury, sun exposure /heat stroke lead to mild endothelial injury, direct endothelial injury by bacterial toxin
Explanation

Answer- A, B, C, D


Question.4

Match the following-

AIIMS PG May-2019
A. Dohle bodies – dilated rough ER
B. Auer rodes – DNA remnants
C. Flame cells – azurophilic
D. Howel jolly body – HbF
Correct Ans: All:Dohle bodies – dilated rough ER, Auer rodes – DNA remnants, Flame cells – azurophilic, Howel jolly body – HbF
Explanation
  1. Dohle bodies-  dilated rough ER
  2. Auer rodes- aurophilic
  3. Flame cells- HbF
  4. Howel jolly body- DNA remnants

Question.5

Which process shows phases of acute inflammation-

AIIMS PG May-2019
A. Apoptosis
B. Pyroptosis
C. Necroptosis
D. Necrosis
Correct Ans: D
Explanation

Answer- D- Necrosis

Direct injury to the endothelium causes cell necrosis and appearance of physical gaps at the sites of detached endothelial cells.

In necrosis phospholipase activation occurs that leads to cell damage and leakage of enzymes outside the cell is followed by inflammatory cells resulting in inflammation.


Question.6

Histologic image of keratinising stratified squamous epithelium in stratum basale layer which of the following cell is not present-

AIIMS PG May-2019
A. Langerhan cells
B. Melanocyte
C. Merkel’s cells
D. Keratocyte
Correct Ans: D
Explanation

Answer- D. Keratocyte

Types of cells found within the stratum basale are melanocytes (pigment-producing cells), Langerhans cells (immune cells), and Merkel cells (touch receptors).


Question.7

All feature of reversible cell injury EXCEPT.

AIIMS PG May-2019
A. ER SWELLING
B. DENS DEPOSITION OF MITOCHONDRIA
C. BLEB
D. Detachment of ribosome
Correct Ans: B
Explanation

Answer- B. DENS DEPOSITION OF MITOCHONDRIA

The ultrastructural charges (seen on electron microscopy) are :-

  1. Plasma membrane alterations ? Blebbing blunting loss of microvilli.
  2. Mitochondrial changes ? Swelling, small amorphous densities.
  3. Dilatation of ER and detachment of ribosome
  4. Nuclear alterations

Question.8

Which of the following Glomerulonephritis has Nephrotic syndrome except.

AIIMS PG May-2019
A. FSGS
B. Post-infectious Glomerulonephritis
C. MPGN
D. Minimal Change Disease
Correct Ans: B
Explanation

Answer- B. Post-infectious Glomerulonephritis

  • Focal segmental glomerulosclerosis is characterised by a sclerosis of segments of some glomerules. It is likely to present as a nephrotic syndrome.
  • Membranous glomerulonephritis may cause either nephrotic or a nephritic syndrome.
  • Post-infectious glomerulonephritis- present with malaise, a slight fever, nausea and a mild nephritic syndrome of moderately increased blood pressure, gross haematuria, and smoky-brown urine.
  • Membranoproliferative GN (MPGN), also known as mesangiocapillary glomerulonephritis- present with the nephritic syndrome, hypocomplementemia.

Question.9

Chimerism phenomenon is associated with which of the following-

AIIMS PG May-2019
A. Paternity test
B. Maternity test
C. Person identification test
D. Organ transplantation case
Correct Ans: D
Explanation

Answer- D. Organ transplantation case

Chimerism can occur in animals is by organ transplantation, giving one individual tissues that developed from a different genome. For example, transplantation of bone marrow often determines the recipient’s ensuing blood type.


Question.10

Correct about regulatory T – cell?

AIIMS PG May-2019
A. Release of granzyme A or B by the TREG which induces apoptosis of target T cells
B. CD 8 associated with suppressive cells
C. CD 4 & CD 25 marker
D. All are true
Correct Ans: C
Explanation

Answer- C. CD 4 & CD 25 marker

Tregs express the biomarkers CD4, FOXP3, and CD25.


Question.11

Example of Dystrophic calcification-

AIIMS PG May-2019
A. Hyperparathyroidism
B. Sarcoidosis
C. Hypervitaminosis D
D. Myostitis ossificans
Correct Ans: D
Explanation

Answer- D. Myostitis ossificans

Myositis Ossificans (MO) is an unusual pathological entity still largely unknown, characterized by dystrophic calcification leading to heterotopic ossification of intramuscular connective tissue.


Question.12

Arrange them in the order of their size of lesions according to depth-

AIIMS PG May-2019
A. Bullous Pemphigoid
B. Epidermolysis Bullosa
C. Pemphigous Vulgaris
D. IgA pemphigus
Correct Ans: All:Bullous Pemphigoid, Epidermolysis Bullosa, Pemphigous Vulgaris, IgA pemphigus
Explanation

Answer- A, B, C, D

  1. Bullous Pemphigoid
  2. Epidermolysis Bullosa
  3. Pemphigous Vulgaris
  4. IgA pemphigus

Question.13

Amyloid protein in Hemodialysis associated with amyloidosis is-

AIIMS PG May-2019
A. AA
B. A ??
C. ??- 2 microglobulin
D. Transthyretin
Correct Ans: C
Explanation

Answer-C. ??– 2 microglobulin

Hemodialysis-associated amyloidosis-

Associated disease is Chronic renal failure

Major fibril protein is A??2m

Clinically related precurosr protein- Beta- 2 microglobulin


Question.14

Under polarised light, the congo red stained amyloid shows-

AIIMS PG May-2019
A. Pink or red color
B. White color
C. Apple green birefringence
D. None
Correct Ans: C
Explanation

Answer- C. Apple green birefringence

Congo red staining shows an apple-green birefringence under polarized light, a diagnostic feature of amyloid.


Question.15

In a patient with Hepatitis B infection. Which one of the markers will be increased?

AIIMS PG May-2019
A. HbsAg
B. HbcAg
C. Anti HbsAg IgG
D. Anti HbcAg IgG
Correct Ans: D
Explanation

Answer- D. Anti HbcAg IgG

  • Best marker for diagnosing acute hepatitis B is IgM anti-HBc as it is found only in the acute phase of hepatitis B (In chronic hepa­titis IgG anti-HBc is found).

Question.16

Medlar bodies are found in-

AIIMS PG May-2019
A. Sporotrichosis
B. Chromoblastomycosis
C. Mycetoma
D. Histoplasmosis
Correct Ans: B
Explanation

Answer- B. Chromoblastomycosis

Medlar bodies, also known as sclerotic or muriform cell.

When present in skin or subcutaneous tissue, the cells are indicative of chromoblastomycosis.


Question.17

Gene which inhibits cell cycle is-

AIIMS PG May-2019
A. p53
B. RB
C. p16
D. Notch receptor
Correct Ans: A
Explanation

Answer-A-p53

Cell cycle inhibitors

  1. CIP/KIP family ? p21, p27, p57
  2. INK4a/ARF family ? p161 NK4a, pl4ARF
  • A G1 arrest can result simply by the p53 induced expression of p21 WAF1/CIP1/Sdi1
  • P53 also down regulates the expression of cyclin A, providing a secondary break on cell cycle progression into the through the S phase.

ENT

Question.1

The following statements about thyroglossal cyst are true, EXCEPT:

AIIMS PG May-2019
A. The cyst is located within 2 cm of the midline
B. Incision and drainage is the treatment of choice
C. The swelling moves upwards on protrusion of tongue
D. It is the frequent cause of anterior midline neck masses in the first decade of life
Correct Ans: B
Explanation

Ans: B.  Incision and drainage is the treatment of choice 

Thyroglossal cyst is a fluid filled sac resulting from a persistent thyroglossal duct.

Excision of the thyroglossal cyst along with the track and the body of the hyoid bone is the treatment of choice (Sistrunk’s operation).

Ref: Problem Based Approach in Pediatric Surgery By Rao, Page 108; Bailey and Love’s Short Practice of Surgery, 24th Edition, Pages 777-78.


Question.2

Female patient came with right side hearing loss, better heard in a noisy environment,Audiogram shown with about 30-40 dB gap between AC-BC of right & left ear. Rinne’s test negative, Weber’s test centralised. Which of the following condition shown?

AIIMS PG May-2019
A. Ménière’s disease
B. stapedial otosclerosis
C. Presbycusis
D. Vestibular schwannoma
Correct Ans: B
Explanation

Answer-B- stapedial otosclerosis

  • In otosclerosis the normal dense endochondral layer of the bony otic capsule gets replaced by irregularly laid spongy bone. Most common site is fissula ante fenestram (anterior to
  • the oval window)
  • Age group affected is 20-45 years (maximum between 20-30 years). Male:female ratio is 1:2.
  • meniere’s  disease i.e Endolymphatic  hydrops     leads  to  SNHL  and  not  conductive  hearing  loss.
  • Tuning ForkTests in Otosclerosis: Rinnes test-negative ,Weber’s test-lateralized to ear with greater conductive loss.

 

Ophthalmology

Question.1

First order neuron of visual pathway

AIIMS PG May-2019
A. Photoreceptor
B. bipolar neuron
C. lateral geniculate body
D. all of the above
Correct Ans: A
Explanation

Answer-:A: photoreceptor

  • Rods and cones are the first-order receptor cells that respond directly to light stimulation.
  • Bipolar neurons are the second-order neurons that relay stimuli from the rods and cones to the ganglion cells.
  • Ganglion cells third-order neurons that form the optic nerve (CN II).


Question.2

IOL placed in the young male., After 10 year what should be done..?

AIIMS PG May-2019
A. IOL should be removed in case of Posterior Capsule ossification [PCO]
B. Never be removed
C. Remove when presbyopia sets in
D. Should be changed after 10 year
Correct Ans: A
Explanation

Answer A. IOL should be removed in case of Posterior Capsule ossification [PCO]

1. The indications for removing an intraocular lens (IOL) are:

  • Chronic uveitis
  • Endothelial corneal dystrophy
  • Uncontrollalbe glaucoma
  • Metal loop cutting pupillary sphincter
  • Gross decentration of IOL (fibrous bands)
  • Extraocular dislocation of IOL G. Recurrent severe hyphema
  • Development of rubeosis iridis
  • Removal of iris (1) Iris tumor (2) Epithelial downgrowth
  • Endophthalmitis
  • Unilateral IOL in pending bilateral aphakia 

2. The following may be indications for removing and/or replacing and/or replacing an IOL:

  • Wrong dioptric power
  • Foreign body attached to IOL
  • IOL covered with pigment
  • Repair retinal detachment after extracapsular cataract extraction
  • Choyce lens too short
  • Dannheim IOL with absorbed supramid loop tips
  • Dislocated Ridley IOL
  • Sclero-conjunctival erosion of Strampelli’s “external-fixation” IOL loop

Referencehttps://www.ncbi.nlm.nih.gov/pubmed/904866


Question.3

The given image is related to following?

AIIMS PG May-2019
A. Marcus Gunn pupil
B. Optic neuritis
C. Robertson-Argyll pupil
D. Pin point pupil
Correct Ans: A
Explanation

Answer-A-Marcus gun pupil

  • RELATIVE AFFERENT PATHWAY DEFECT (RAPD) OR MARCUS GUNN PUPIL It is the paradoxical response of a pupil to light in the presence of a relative afferent pathway defect (RAPD). Caused by an incomplete optic nerve lesion or a severe retinal disease. It is best tested by ‘swinging flashlight test’.
  • Normally, both pupils constrict equally and the pupil to which light is transferred remains tightly constricted.
  • In the presence of RAPD in one eye, the affected pupil will dilate (paradoxical response) when the flashlight is moved from the normal eye to the abnormal eye.
  • This response is called Marcus Gunn Pupil.
  • It is the earliest indication of optic nerve disease even in the presence of a normal visual acuity

Question.4

The Following test is used for – 

AIIMS PG May-2019
A. Streopsis Test
B. Color VIsion
C. Strabismus
D. All of the above
Correct Ans: A
Explanation

Answer A. Streopsis Test

  • Stereopsis is the ability to perceive depth and 3- d structure obtained on the basis of visual information deriving from two eyes by individual with normally developed binocular vision stereopsis is usually reserved for the impression of depth arising from binocular disparity
  • stereopsis is measured at near as part of clinical assessment of patients disorders of ocular motility and strabismus • stereopsis can be measured near and distance

Question.5

on examination 18 month old child has inward deviation of both eyelids,which of the  following test should be  done?

AIIMS PG May-2019
A. Forced duction test
B. Cover Uncover Test
C. Fundus Examination
D. All of the above
Correct Ans: B
Explanation

Answer- B- Cover Uncover Test

Since the diagnosis is strabismus the correct answer is B

The forced duction test is performed in order to determine whether the absence of movement of the eye is due to a neurological disorder or a mechanical restriction. The anesthetized conjunctiva is grasped with forceps and an attempt is made to move the eyeball in the direction where the movement is restricted.

Since the diagnosis is strabismus the correct answer is B


Question.6

A patient  with a history  of  diabetes  for  one  year with no other complications  should have  an  ophthalmic  examination?

AIIMS PG May-2019
A. As early as feasible
B. After 5 years
C. After 10 years
D. Only after visual symptoms level
Correct Ans: A
Explanation

Answer– A i.e.  As  early  as  feasible

  • All  diabetic  (IDDM  & NIDDM both)  aged  over 12  years  and/or entering puberty  should  be  screened (visual activity measurement  and fundus examination by  ophthalmoscopy)
  • For retinopathy. and those  with risk  for visual loss  referred  to  an  ophthalmologist.
  • Type I  DM  (IDDM)  require  ophthalmoscopic  examination within  3  years of diagnosis  and annual  review.  (If  lt is  diagnosed  before  the  age  of puberty).
  • Type II  DM (NIDDM)  require  ophthalmoscopic  examination  at the  time of  diagnosis (because  it  is usually  diagnosed after  the  age  of 12 years)  and annual  review.

Question.7

Image nail piercing into the cornea of the eye which of the following is not done at first:

AIIMS PG May-2019
A. primary survey
B. testing visual acuity
C. removal of foriegn body
D. antibiotics
Correct Ans: C
Explanation

Answer C. removal of foriegn body

Removal of The nail should not be done at first rather the patient should be immediately referred to an ophthalmologist for further management.

Referencehttps://www.ncbi.nlm.nih.gov/pmc/articles/PMC4790160/

Preventive & Social Medicine

Question.1

Government initiative to improve the facilities in labour room in all govt hospitals is under

AIIMS PG May-2019
A. Ayushman Bharat Scheme
B. Laqshya
C. Newborn delivery room program
D. Janani suraksha yojana
Correct Ans: B
Explanation

Ans.B. Laqshya

‘LaQshya’ programme of the Ministry of Health and Family Welfare aims at improving quality of care in labour room and maternity Operation Theatre (OT).

Objective: 

  • To reduce maternal and newborn mortality & morbidity due to APH, PPH, retained placenta, preterm, preeclampsia & eclampsia, obstructed labour, puerperal sepsis, newborn asphyxia, and sepsis, etc.
  • To improve Quality of care during the delivery and immediate post-partum care, stabilization of complications and ensure timely referrals, and enable an effective two-way follow-up system.
  • To enhance satisfaction of beneficiaries visiting the health facilities and provide Respectful Maternity Care (RMC) to all pregnant women attending the public health facility.

Question.2

Which of the following are the responsibilities of ASHA worker ?

AIIMS PG May-2019
A. Dot Providers of Directly Observed Treatment Short-course
B. Accompanying the pregnant to the Labour room
C. To generate demand on key determinants of health
D. All of the above
Correct Ans: D
Explanation

Ans. A. B. C. 

ROLES OF ASHA WORKERS:

  • ASHA create awareness and provide information to the community on determinants of health such as proper diet and nutrition, basic sanitation and hygienic practices, healthy living and working conditions, information on existing health services and the need for timely utilization of health and family welfare services at doorsteps.
  • Conduct home visits of the pregnant women/mother/newborn under Home Based Post Natal Care (HBPNC), and they are supposed to counsel pregnant women on birth preparedness, importance of safe delivery, breastfeeding and complementary feeding, immunization, contraception and prevention of common infections including Reproductive Tract Infection/Sexually Transmitted Infection (RTIs/STIs) and care of the young child.
  • Mobilize the community and facilitate them in accessing health and health related services available at the village/sub-center and other public health facilities, such as Routine Immunization (RI), Ante Natal Check-ups (ANCs), Post Natal Check-ups (PNCs), sanitation and other services being provided by the Government.  ASHA have to work with the Village Health Sanitation Nutrition Committee/Village Level Committee (VHSNC/VLC) of the Gram Panchayat to facilitate a comprehensive village health plan with ANM, AWWs and PRI members.
  • Mobilize targeted community once in a month for the celebration of Village Health Nutrition Days (VHND) at their Aanganwadi Centre.
  • ASHA have to arrange escort/accompany pregnant women & children requiring treatment/ admission to the nearest pre- identified health facility i.e. Primary Health Centre/Community Health Centre/ First Referral Unit (PHC/CHC /FRU).
  • ASHA have to provide primary medical care for minor ailments such as diarrhoea, fevers, and first aid for minor injuries.
  • Dot Providers of Directly Observed Treatment Short-course (DOTS) under Revised National Tuberculosis Control Programme (RNTCP).
  • ASHA are also acting as a Depot Holder for essential provisions being made available to every habitation like Oral Rehydration Therapy (ORS), Iron Folic Acid Tablet (IFA), chloroquine, Disposable Delivery Kits (DDK), Contraceptives (Condoms, Oral Pills, Emergency Pills), etc. Provision of Drug Kit and HBPNC Kit has been made for ASHA. Contents of the Drug/HBPNC Kits are based on the recommendations of the expert/technical advisory group set up by the Govt of India.
  • ASHA are expected to provide first information about the births and deaths in her village and any unusual health problems/disease outbreaks in the community to the SC/PHC/CHC or directly to the District Authorities or even to the State HQ at the NRHM Help Line.

Question.3

Biomedical waste management in Match the following:

  1. Chemical waste
  2. Syringe Caps
  3. Scalpel
  4. Glasswares
  5. Gloves
AIIMS PG May-2019
A. Category No.4
B. Category No.6
C. Category No.7
D. Category No.10
Correct Ans: All:Category No.4, Category No.6, Category No.7, Category No.10
Explanation

Answer: D. IV. C. III A. I B. II 

HOSPITAL WASTE CATEGORIES: 

Categories of bio-medical waste in India
1 Human anatomical waste
2 Animal waste
3 Microbiology and biotechnology waste
4 Waste sharps (used and unused needles, syringes, scalpels etc)
5 Cytotoxic drugs and discarded medicines
6 Solid waste (contaminated with blood and fluids including cotton, dressings, soiled plasters, beddings)
7 Solid waste (other than waste sharps such as rubber tubes, catheters and I.V sets)
8 Liquid waste
9 Incineration ash
10 Chemicals used in disinfection, insecticides

 COLOR CODING & TYPES OF CONTAINER:

Colour coding Type of Container Waste Category Treatment options
Yellow Plastic Bags Human and animal wastes, Microbial and Biological wastes and soiled wastes

(Cat 1,2,3 and 6)

Incineration/ Deep Burial
Red Disinfected container/ Plastic bags Microbiological and Biological wastes, Soiled wastes, Solid wastes

(Cat 3,6,7)

Autoclave/ Microwave/ Chemical Treatment)
Blue/ White/ Transparent Plastic bag, Puncture proof container Waste sharps and solid waste

( Cat 4 &7)

 

Autoclave/ Microwave/ Chemical Treatment Destruction and Shredding
Black Plastic bag Discarded medicines, Cytotoxic drugs, Incineration ash and chemical waste

(Cat 5,9 & 10)

Disposal in secured land fills
Green Plastic Container General waste such as office waste, food waste & garden waste Disposed in secured landfills

Question.4

Immunization that could help post disaster includes –

AIIMS PG May-2019
A. Measles
B. Cholera
C. Typhoid
D. All
Correct Ans: A
Explanation

Ans. A. Measles

Vaccines recommended in disasters

  • Following vaccines are recommended
  1. Children < 10 years :- DPT, inactivated polio (IPV), H.influenzae type b (Hib), hepatitis B, pneumococcal conjugate vaccine (PCV), measles-mumps-rubella (MMR), varicella vaccine, influenza, hepatitis A and rotavirus.
  2. Children and adolescents (11-18 years):- Tetanus, diphtheria, pertussis, meningococcal conjugate vaccine (MCV), Influenza.
  3. Adults (>18 years):- Tetanus, diphtheria, pertussis, pneumococcal polysaccharide vaccine (PPSV23), and influenza.
  • Vaccination against typhoid and cholera is not recommended.

Question.5

In a village of 20,000 population 456 births occurred in april, 56 showed no signs of life at birth, 56 died before 28 days of life, 34 died between 28 days- 1 year, 500 was total number of deaths in that year. Infant mortality rate?

AIIMS PG May-2019
A. 197.4
B. 320
C. 225
D. 125
Correct Ans: C
Explanation

Ans.C.225

IMR = Number of deaths of children less than 1 year of age in a year X 1000/number of live births in the same year.

IMR=56+34 X1000/400= 225


Question.6

A Study conducted in a population.

Diastolic Blood Pressure mean 110 mm Hg with SD of 11 mm Hg, Vitamin D mean 18 ng/ml with SD of 3 ng/ml. what is the relation between the two variance

AIIMS PG May-2019
A. Variance of Vitamin D > DBP
B. Variance of DBP > 4 times Vitamin D
C. Data insufficient to comment upon
D. None
Correct Ans: B
Explanation

Ans.b). Variance of DBP > 4 times Vitamin D

  • Coefficient of Variance (CV) is a tool to compare variability of two different characteristics (eg. BP, serum creatinine, height weight etc.) in the same group of subjects or compare variability of the same character in two or more different groups.
  • Thus it is a measure used to compare relative variability.
  • For example, coefficient of variance measures
  1. whether weight varies more than height in a group of student
  2. whether weight varies more in girls of boys.

Calculation:

  • CV=  standard deviation x 100/ mean

For Blood pressure- 110×100/11=1000

For Vitamin D- 3X100/18= 16.7


Question.7

Broken ampula is thrown into which coloured bin?

AIIMS PG May-2019
A. Red
B. Yellow
C. Blue
D. Black
Correct Ans: C
Explanation

Ans. C. Blue

COLOR CODING & TYPES OF CONTAINER: 

Colour coding Type of Container Waste Category Treatment options
Yellow Plastic Bags Human and animal wastes, Microbial and Biological wastes and soiled wastes

(Cat 1,2,3 and 6)

Incineration/ Deep Burial
Red Disinfected container/ Plastic bags Microbiological and Biological wastes, Soiled wastes, Solid wastes

(Cat 3,6,7)

Autoclave/ Microwave/ Chemical Treatment)
Blue/ White/ Transparent Plastic bag, Puncture proof container Waste sharps and solid waste

( Cat 4 &7)

Autoclave/ Microwave/ Chemical Treatment Destruction and Shredding
Black Plastic bag Discarded medicines, Cytotoxic drugs, Incineration ash and chemical waste

(Cat 5,9 & 10)

Disposal in secured land fills
Green Plastic Container General waste such as office waste, food waste & garden waste Disposed in secured landfills

Question.8

Case control study was done regarding breast cancer & risk factors & odds ratio was obtained. Which of the following are causative factors ?

Parameter Case Control OR
Age at menarche>12 1714 2329 1.04
Menopause 1771 2408 1.53
OCP use 935 1105 1.02
Smoking 93 214 0.97
Family history 513 502 1.10
BMI>27.5 165 493 0.53
Breastfeeding 942 2514 0.56
Multiparity(2) 2778 3366 1.05
AIIMS PG May-2019
A. 6
B. 5
C. 3
D. 2
Correct Ans: B
Explanation

Ans.B) 5

  • An odds ratio (OR) is a statistic that quantifies the strength of the association between two events, A and B.
  • Two events are independent if and only if the OR equals 1: the odds of one event are the same in either the presence or absence of the other event.
  • If the OR is greater than 1, then A and B are associated (correlated) in the sense that, compared to the absence of B, the presence of B raises the odds of A, and symmetrically the presence of A raises the odds of B.
  • Conversely, if the OR is less than 1, then A and B are negatively correlated, and the presence of one event reduces the odds of the other event.
  • Most Importantly Odds ratio does not demonstrate causality .
  • But we can say that they are positively correlated. Since 5 Factors have odds ratio more than 1 so the answer will be B

Question.9

A study is to be conducted with regards to the fat content in the expressed breast milk of pre-term infants as compared to term infants. Which study design is best suited?

AIIMS PG May-2019
A. Case control
B. Prospective cohort
C. Longitudinal study
D. Ambispective
Correct Ans: A
Explanation

Ans. A. Since there is a comparison between fat content of EBM in preterm vs term infant the best study design will be, Case control.

Prospective Cohort study will be appropriate if we want to see the change in the fat content of EBM in preterm infants.


Question.10

Key indicator for AFP surveillance -.

AIIMS PG May-2019
A. At least one case of non-polio AFP per year per 1000 population of under 5 years
B. At least one case of non-polio AFP per year per 100000 population of under 5 year
C. At least one case of non-polio AFP per year per 1000 population of under 15 years
D. At least one case of non-polio AFP per year per 100000 population of under 15 years
Correct Ans: D
Explanation

Ans.d) At least one case of non-polio AFP per year per 100000 population of under 15 years 

The number of AFP cases reported each year is used as an indicator of a country’s ability to detect polio, even in countries where the disease no longer occurs.

Polio surveillance

  • It is the most important part of whole polio eradication intiative. It has two components:-
  • Acute flaccid paralysis (AFP) surveillance
  • Acute flaccid paralysis is defined as acute onset (< 4 weeks) of flaccid paralysis (reduced tone) without other obvious cause in children WHO recommends the immediate reporting and investigation of every case of AFP in children less than 15 years.

Question.11

A graph given. Calculate regression equation.

AIIMS PG May-2019
A. X = 2Y + X
B. X = Y + 2*X
C. X = Y/2
D. Y = X/2
Correct Ans: C
Explanation

Answer C. X = Y/2

The Values of X, Y are

5,10

10,20

15, 30

20,40 ..

So The Correct Regression equation is X = Y/2


Question.12

In assessing the association between maternal nutritional status and the birth weight of the newborns, two investigators A and B studied separately and found significant results with p values 0.02 and 0.04 respectively. From this information, what can you infer about the magnitudes of association found by the two investigators –

AIIMS PG May-2019
A. The magnitude of association found by Investigator A is more than found by B
B. The magnitude of association found by Investigator B is more than found by A
C. The estimates of association obtained by A and B will be equal, since both are significant
D. Nothing can be concluded as the information given is inadequate
Correct Ans: D
Explanation

Ans. D. Nothing can be concluded as the information given is inadequate 


Question.13

Looking at the scatter diagram below What is the relation between two graphs?

AIIMS PG May-2019
A. Positive correlation between birth weight and gestational age
B. Negative correlation between birth weight and gestational age
C. No correlation between birth weight and gestational age
D. The data is insufficient to comment
Correct Ans: A
Explanation

Ans. A. Positive correlation between birth weight and gestational age

Scatter plot of bone SOS vs gestational age, r=0.569, p<0.001 (y=2161+26.2x). (b) Scatter plot of bone SOS vs birth weight, r=0.420, p<0.001 (y=2800+0.113x).

This shows a significant (p=0.001) positive correlations between SOS and gestational age, birth weight, length, head circumference and tibial length.


Question.14

Matching is method to eliminate confounding bias. It is used in-

AIIMS PG May-2019
A. Case control study
B. Cohort study
C. Experimental study
D. Cross sectional study
Correct Ans: A
Explanation

Ans. A. Case control study 

  • Matching is most useful in case control studies (though it can also be used in cohort studies, but benefit is not assured if exposure is not randomized).

Method Used to Control Confounding

Method Utility to control confounding
Randomization Most ideal method
Restriction Limiting study to people who have particular characteristics
Matching Mostly useful in case control studies
Stratification Useful for larger studies
Statistical modelling When many confounding variables exist simultaneously

Question.15

Test used to assess quantitative observations before and after an intervention –

AIIMS PG May-2019
A. Unpaired T-test
B. Paired T-test
C. Chi-square test
D. Fisher-T-test
Correct Ans: B
Explanation

Ans. B.Paired T-test

Paired test : Is applied to paired data, when each individual gives a pair of observations such as : when observations are made before and after the play of a factor e.g. pulse rate before and after a drug. Further, it proceeds similar to the unpaired test.


Question.16

Transmission assessment survey  (TAS) is done in the following?

AIIMS PG May-2019
A. To determine when infections have been reduced below these target thresholds
B. For assessing primary immunization coverage
C. To provide reliable estimates of birth rate, death rate and infant mortality rate
D. All
Correct Ans: A
Explanation

Ans. A. To determine when infections have been reduced below these target thresholds

  • WHO recommends the transmission assessment survey (TAS) to determine when infections have been reduced below these target thresholds and MDA can stop.
  • Once MDA has stopped, TAS is used as a surveillance tool to determine that infection levels are sustained below target thresholds.
  • Mass drug administration (MDA) is needed to reduce infection in the community to levels below a threshold at which mosquitoes are unable to continue spreading the parasites from person to person and new infections are prevented.

Skin

Question.1

After sequential arrangement of blistering disorder starting from superficial to deep blisters, which is the deepest among these?

AIIMS PG May-2019
A. IgA pemphigus
B. EBD
C. PV
D. Bullous pemphigoid+
Correct Ans: C
Explanation

Ans. C. PV

  • Pemphigus vulgaris (PV) is a rare and serious (potentially life-threatening) condition that causes painful blisters to develop on the skin and lining of the mouth, nose, throat and genitals.
  • The blisters are fragile and can easily burst open, leaving areas of raw unhealed skin that are very painful and can put you at risk of infection.
  • H&E: Subepidermal bullae

  • DIF: linear IgG and C3 deposition at the basement membrane zone

 


Question.2

Identify the condition below?

AIIMS PG May-2019
A. Tardieu spots
B. ITP
C. Vasculitis
D. DIC spots
Correct Ans: A
Explanation

Ans. A. Tardieu spots

  • Asphyxial petechial haemorrhages do occur on the serosal surfaces of internal organs eg.heart and lung as shown in the pictures.
  • They were first described by Tardieu in the 19th century and were named after him as tardieu spots.
  • It were considered at that time and for a long period after as pathognomonic  sign of asphyxial deaths.
  • Such petechial haemorrhages do occur in deaths related  to infections,blood coagulopathy disorders and micro emboli.
  • It can also occur due to rupture of engorged capillaries in the dependent parts.
  • It can also be noticed in some cases of natural deaths, so it is by no means are pathognomonic of asphyxial deaths.

Question.3

Identify the condition below and which of the following is its most characteristic microscopic finding?

AIIMS PG May-2019
A. Intracellular inclusion bodies
B. Multinucleated cells
C. Subepithelial Bullae
D. Councilman body
Correct Ans: A
Explanation

Ans. A. Intracellular inclusion bodies

(Ref: Robbins 9/e p1176; Fitzpatrick 6/e p2347)

  • All findings are highly suggestive of Molluscum contagiosum, which is caused by the poxvirus.

Molluscum contagiosum:

  • On microscopic examination, lesions show cuplike verrucous epidermal hyperplasia. The diagnostically specific structure is the molluscum body, which occurs as a large, ellipsoid, homogeneous, cytoplasmic inclusion in cells of the stratum granulosum and the stratum corneum.
  • In H&E stain, these inclusions are eosinophilic in the blue-purple stratum granulosum and acquire a pale blue hue in the red stratum corneum.
  • Numerous virions are present within molluscum bodies.


Question.4

Image shown Child having vesicular lesions on lower lip, palms & soles, question was asked which of the following group causes the disease?

AIIMS PG May-2019
A. HSV – 1
B. Picornaviridae
C. Togaviridae
D. Pox virus
Correct Ans: C
Explanation

Ans. C. Togaviridae

  • Vesiculobullous lesions were common cutaneous manifestation of chikungunya fever in infants.
  • Pigmentary lesions were generalized, brownish black and predominantly involved face and extremities.
  • Generalized maculopapular eruption or erythema with islands of normal skin were noted, mostly 2-3 days after onset of fever.
  • It started on trunk and then spread centrifugally involving face, palms and soles.
  • Mucosal regions were spared.
  • Mostly it subsided in 4-5 days, but in 5 cases there was peeling of the skin over body, palms and soles, resembling staphylococcal scalded skin syndrome (SSSS).

Question.5

Which of the following cells not present in stratum basale layer?

AIIMS PG May-2019
A. Langerhans cells
B. Merkel cells
C. Keratocyte
D. Melanocytes
Correct Ans: A
Explanation

Ans. A. Langerhans cells

  • The stratum basale is primarily made up of basal keratinocyte stem cells, which can be considered the stem cells of the epidermis.
  • They divide to form the keratinocytes of the stratum spinosum, which migrate superficially.
  • Other types of cells found within the stratum basale are melanocytes (pigment-producing cells), and Merkel cells (touch receptors).

Psychiatry

Question.1

Which one of the following disorders is related to sense of unreality?

AIIMS PG May-2019
A. Depersonalization disorder
B. Derealization disorder
C. Delusion
D. Phobias
Correct Ans: A
Explanation

Ans. A. Depersonalization disorder

Depersonalization:

  • Depersonalization is a sense of unreality or strangeness concerning the self, manifested by feeling detached from and being an outside observer of one’s mental processes or body.
    • A patient who experiences depersonalization may describe feeling like things “I have stepped outside of myself and am watching myself doing things”.

Derealization:

  • Derealization  refers to feeling detached from one’s environment so that the sense  of the reality of the external world is lost.
  • E.g, I feel as if everything is unreal and those around me are actors in a play or two dimensional cardboard figures.

Question.2

Which of the following is not included in the grades of insight?

AIIMS PG May-2019
A. Intellectual
B. Emotional
C. Psychological
D. Judgemental
Correct Ans: D
Explanation

Ans. D. Judgemental

  • Neziroglu and Stevens Proposed four different levels of insights:
    • True emotional insight.
    • Intellectual insight
    • Partial internally and externally based insight
    • Denial Of Illness.
  • True emotional insight Is representative of the highest level of insight possible.
  • In it the patients’ awareness and understanding Of their own thoughts, feelings and motives can be used to change behavior.

Question.3

Which of the following dementia is associated with visual hallucinations?

AIIMS PG May-2019
A. Lewy body dementia
B. AIDS related Dementia
C. Huntington’s disease
D. Mixed dementia
Correct Ans: A
Explanation

Ans. A.  Lewy body dementia –

  • Lewy body dementia, also known as dementia withLewy bodies, is the second most common type of progressive dementia after Alzheimer’s disease dementia.
  • Protein deposits, called Lewy bodies, develop in nerve cells in the brain regions involved in thinking, memory and movement (motor control).

Radiology

Anaesthesia

Question.1

Action of proparacaine starts within?

AIIMS PG May-2019
A. 2 minutes
B. 5 minutes
C. 15 minutes
D. 20 minutes
Correct Ans: A
Explanation

Ans. A. 2 minutes

  • Proparacaine and tetracaine are indicated to produce local anesthesia of short duration for ophthalmic procedures including measurement of intraocular pressure (tonometry), removal of foreign bodies and sutures, and conjunctival and corneal scraping in diagnosis and gonioscopy.
  • Onset of action:
    • Proparacaine—Within 20 seconds.
    • Tetracaine—Approximately 15 seconds.
  • Duration of action:
    • Proparacaine—15 minutes or longer.
    • Tetracaine—10 to 20 minutes; average 15 minutes.

Internal Medicine

Question.1

Young patient with fever, arthralgia,headache and is on NSAIDs,and has macules over the nose image is given-

AIIMS PG May-2019
A. Chikungunya
B. Measles
C. Dengue
D. Fixed drug reactions
Correct Ans: A
Explanation

Answer- A. Chikungunya

Symptoms of the disease include:

  • Fever
  • A petechial or maculopapular rash of the trunk and occasionally the limbs
  • Arthralgia or arthritis affecting multiple joints.
  • The pain associated with CHIKV infection of the joints persists for weeks or months.
  • Typical macules over the nose.
  • Typically, the fever lasts for two days and then ends abruptly.
  • Ocular inflammation may present as iridocyclitis and have retinal lesions as well.
  • Pedal edema (swelling of legs) is observed in many patients.

Question.2

Which score is used for wound infection-

AIIMS PG May-2019
A. Glascow coma scale
B. Southampton score
C. Apgar score
D. SIRS score
Correct Ans: B
Explanation

Answer- B. Southampton score

Southampton score and ASEPSIS criteria are used for wound infection.


Question.3

causes of thyroid storm

AIIMS PG May-2019
A. ineffective pre-operative hormonal control
B. manhandling of the thyroid gland during surgery
C. parathyroid tail left after surgery
D. Poor Preoperative preparation of the patient
Correct Ans: D
Explanation

Answer- D. Poor Preoperative preparation of the patient

  • Commonest reason for thyroid storm is poor pre operative preparation of the patient.
  • Acute illness (stroke, infection, trauma, diabetic ketoacidosis)
  • Surgery
  • Inadequate control of hyperthyroidism
  • Radioactive treatment of hyperthyroidism

Question.4

If the cycle starts with ‘ a ‘ wave of jvp . Arrange the following in order-

AIIMS PG May-2019
A. R wave
B. T wave
C. 1st heart sound
D. Rapid ejection phase
Correct Ans: All:R wave, T wave, 1st heart sound, Rapid ejection phase
Explanation

Answer- A, B, C, D

The “a” wave is approximately synchronous with the first heart sound (S1) and just precedes the carotid upstroke.

Shortly after the a-wave there is a second peak , the c-wave.  The c-wave immediately follows the r wave of the ECG waveform.

The v-wave corresponds to the end of the t wave in the ECG waveform.


Question.5

Which of the following doesn’t have any effect on pancreatic secretion?

AIIMS PG May-2019
A. Cck
B. Gastrin
C. Secretin
D. Gastric inhibitory polypeptide
Correct Ans: D
Explanation

Answer- D. Gastric inhibitory polypeptide

CCK-  As chyme floods into the small intestine, cholecystokinin is released into blood and binds to receptors on pancreatic acinar cells, ordering them to secrete large quantities of digestive enzymes.

Secretin-

  • The predominant effect of secretin on the pancreas is to stimulate duct cells to secrete water and bicarbonate.
  • Gastrin- Stimulate acid secretion by the parietal cell, gastrin stimulates pancreatic acinar cells to secrete digestive enzymes.

Question.6

A young man was on high protein diet. After 3 days he developed weakness. Blood investigation revealed hypoglycemia. Hypoglycemia is due to inhibition of which of the following enzymes-

AIIMS PG May-2019
A. Glucose 6 phosphatase
B. Glycogen phosphorylase
C. Phosphoglucomutase
D. PEP carboxylase
Correct Ans: D
Explanation

Answer- D. PEP carboxylase


Question.7

What is the most common tumor of mediastinum?

AIIMS PG May-2019
A. Neurogenic
B. Thymoma
C. Lymphoma
D. Metastatic tumor
Correct Ans: A
Explanation

Answer- A. Neurogenic

the most common mediastinal masses are neurogenic tumors (23%), thymomas (21%), lymphomas (13%) and germ cell tumors (12%). Mediastinal masses are most frequently located in the anterosuperior mediastinum (54%), with the posterior (26%) and middle mediastinum (20%) being less frequently involved


Question.8

In which of the following tumor MRI is better than CT Scan..

AIIMS PG May-2019
A. Brochogenic Carcinoma
B. SCC of lung
C. Mesothelioma
D. Pancost tumor
Correct Ans: D
Explanation

Answer- D. Pancost tumor

MRI: An MRI is generally more accurate than a CT scan at uncovering the extent to which a tumor has invaded other structures


Question.9

Which of the following has intracellular receptor –

AIIMS PG May-2019
A. Glucagon
B. Insulin
C. Epinephrine
D. Thyroxine
Correct Ans: D
Explanation

Answer- D. Thyroxine

Classic hormones that use intracellular receptors include thyroid and steroid hormones.


Question.10

A patient presents to the ER after a RTA with multiple rib injuries. He is conscious, speaking single words. RR = 40/min, BP= 90/40 mmHg. What is the next immediate step in management?

AIIMS PG May-2019
A. Intubate the patient
B. Urgent fluid infusion
C. Chest X ray
D. Needle insertion in 2nd ICS
Correct Ans: D
Explanation

Answer- D. Needle insertion in 2nd ICS

This is a case of tension pneumothorax. Although the new ATLS update is 5th intercostal space in mid maxillary line, but in this question we will go with a time tested method of needle in 2nd intercostal space.


Question.11

Name the 2nd and 3rd marked area in the diagram-

AIIMS PG May-2019
A. Pulmonic area & Tricuspid area
B. Erb’s point and Pulmonic area
C. Tricuspid area & Mitral area
D. Aortic area & Mitral area
Correct Ans: C
Explanation

Answer- C. Tricuspid area & Mitral area


Question.12

Identify the disease in the image?

AIIMS PG May-2019
A. Cutaneous larva margins
B. Scabies
C. Psoriasis
D. None
Correct Ans: A
Explanation

Answer- A. Cutaneous larva margins

The most common species causing this disease is Ancylostoma braziliense.


Question.13

A patient with Hb 7 was to be transfused with 2 packs of blood. The first pack was transfused in 2 hours after which vitals were stable and then the transfusion of the next pack was started . But suddenly the patient develops breathlessness and hypertension. What can be the cause of this sudden reaction?

AIIMS PG May-2019
A. transfusion related circulatory overload (TACO)
B. allergic reaction to transfused blood
C. transfusion related acute lung injury (TRALI)
D. transfusion related acute renal failure
Correct Ans: C
Explanation

Answer- C. transfusion related acute lung injury (TRALI)

Transfusion-related acute lung injury (TRALI) is a serious blood transfusion complication characterized by the acute onset of non-cardiogenic pulmonary edema following transfusion of blood products.

Hypotension, and fever that develop within 6 hours after transfusion and usually resolve with supportive care within 48 to 96 hours. Although hypotension is considered one of the important signs in diagnosing TRALI, hypertension can occur in some cases.


Question.14

The severity of mitral stenosis can be judged by-

AIIMS PG May-2019
A. Intensity of murmur
B. Duration of murmur
C. Left ventricular S3
D. Loud S1
Correct Ans: B
Explanation

Answer- B. Duration of murmur

Duration depends on severity of MS.

In severe MS, the mid diastolic murmur is long and merges with the presystolic murmur to produce holodiastolic murmur..

Surgery

Question.1

Which of the following does not cause head and neck squamous cell carcinoma

AIIMS PG May-2019
A. Alcohol
B. Betel nut
C. HPV
D. EBV
Correct Ans: D
Explanation

Answer- D .EBV

The most important risk factors for developing HNSCC are tobacco smoking and alcohol consumption.

Consumed in betel quids containing areca nut  increases the risk of developing HNSCC.

HNSCCs of the oral cavity and oropharynx, are becoming more prevalent, which may be related to an increase in oral and oropharyngeal HPV infection.


Question.2

Most common cause of intertrochanteric fracture in old is

AIIMS PG May-2019
A. Osteoporosis
B. Paget’s dis
C. Osteopetrosis
D. Osteomalacia
Correct Ans: A
Explanation

Answer- A. Osteoporosis

  • Having osteoporosis
  • Having a history of other bone problems or fractures
  • Having low bone density and low muscle mass

Question.3

Numbness on shaving after parotidectomy due to which nerve injury?

AIIMS PG May-2019
A. Facial nerve
B. Auriculotemporal nerve
C. Greater auricular nerve
D. Occipital nerve
Correct Ans: B
Explanation

Answer- B. Auriculotemporal nerve


Question.4

Most common functional tumors of endocrine pancreas?

AIIMS PG May-2019
A. Vipoma
B. Gastrinoma
C. Glucagonoma
D. Somatostatinoma
Correct Ans: B
Explanation

Answer- B. Gastrinoma

Insulinoma is the most common functional tumour but if that wasnt there in the optios then gastrinoma is the next answer.


Question.5

Chronic alcoholic comes with pain epigastrium and recurrent vomiting. On examination guarding in upper epigastrium. Chest X-ray normal. What to do next?

AIIMS PG May-2019
A. UGI endoscopy
B. Serum lipase
C. CECT
D. Alcohol breath test
Correct Ans: B
Explanation

Answer- B. Serum lipase

Serum amylase and lipases are the initial investigations done in patients with acute pancreatitis.

CECT is the investigation of choice but the initial investigation in such patients.


Question.6

During diagnostic laparoscopy for undescended testis, there are absent testicular vessels. What should be done next?

AIIMS PG May-2019
A. Explore further
B. Nothing is to be done
C. Inguinal exploration
D. Scrotal examination
Correct Ans: B
Explanation

Answer- B. Nothing is to be done

Nothing is to be done. Absent testicular vessels denotes that there has been intra- uterine torsion and in that case further exploration is required.


Question.7

Which of the following structure is not removed in radical neck dissection-

AIIMS PG May-2019
A. Sublingual
B. Submandibular
C. Tail of parotid
D. Level 2 b lymph nodes
Correct Ans: A
Explanation

Answer- A. Sublingual

Structures removed during radical neck dissection-

  • Level 1-5 In
  • Sternocleidomastoid, IJV, spinal accessory nerve
  • Submandibular gland
  • Tail of parotid

Question.8

All are features of SIRS except-

AIIMS PG May-2019
A. RR> 24 & Paco2 <22mm hg
B. WBC >11 or <4
C. Temperature <36 and >38
D. PR >90
Correct Ans: Select
Explanation

Answer- A. RR> 24 & Paco2 <22mm hg


Question.9

Treatment of choice in a patient with a staghorn calculus with mild hydronephrosis?

AIIMS PG May-2019
A. ESWL
B. PCNL
C. RIRS
D. Open surgery
Correct Ans: B
Explanation

Answer- B. PCNL

PCNL is the best answer as the patient has hydronephrosis and in an obstructed system ESWL is not useful as stone fragments don’t get cleared.


Question.10

A patient presents to the ER after a RTA with multiple rib injuries. He is conscious, speaking single words.

RR= 40/ minute, BP= 90/40mm Hg. What is the next immediate step in management?

AIIMS PG May-2019
A. Intubate the patient
B. Urgent fluid infusion
C. Chest X- ray
D. Needle insertion in 2nd ICS
Correct Ans: D
Explanation

Answer- D. Needle insertion in 2nd ICS

This is a case of pneumothorax. Although the new ATLS update is 5th intercostal space in mid axillary line but in this question we will go with a time tested method of needle in the 2nd ICS.


Question.11

Percentage of SSI rate in patients with a clean contaminated wound?

AIIMS PG May-2019
A. 1-2%
B. <10%
C. 10- 20 %
D. 20- 30 %
Correct Ans: B
Explanation

Answer- B. <10%

  • Clean wound- 1- 2%
  • Clean contaminated- 7- 10%
  • Contaminated- 10- 20%
  • Dirty- >20%


Question.12

Which one of the following does not classify as locally advanced breast cancer?

AIIMS PG May-2019
A. Tumour more than 4cm
B. Inflammatory breast cancer
C. Chest wall involvement
D. Skin involvement
Correct Ans: A
Explanation

Answer- A. Tumour more than 4cm

Locally advanced breast cancer-

T4 N2 N3 M0


Question.13

Which of the following is not a component of quick SOFA (qSOFA) scoring?

AIIMS PG May-2019
A. Bilateral undilated pupils
B. Altered Mentation
C. Glasgow Coma Score
D. SBP <= 100 mm Hg
Correct Ans: A
Explanation

Answer- A. Bilateral undilated pupils

Assessment qSOFA score
Low blood pressure (SBP = 100 mmHg) 1
High respiratory rate (= 22 breaths/min) 1
Altered mentation (GCS = 14) 1

Question.14

Which of the following would you consider orthostatic hypotension?

AIIMS PG May-2019
A. SBP fall by 20 mm hg, DBP fall by 10 mm hg within 3 minutes
B. SBP fall by 20 mm hg, DBP fall by 10 mm Hg within 6 minutes
C. SBP fall by 30 mm hg, DBP fall by 20 mm Hg within 3 minutes
D. SBP fall by 30 mm hg, DBP fall by 20 mm hg within 6 minutes
Correct Ans: A
Explanation

Answer- A. SBP fall by 20 mm hg, DBP fall by 10 mm hg within 3 minutes

Orthostatic hypotension is defined as a decrease in systolic blood pressure of 20 mm Hg or a decrease in diastolic blood pressure of 10 mm Hg within three minutes of standing when compared with blood pressure from the sitting or supine position.


Question.15

Identify the image of the instrument-

AIIMS PG May-2019
A. Bent Hohmann Retractors
B. Bone Holding and Reduction Clamps and Forceps
C. Bone cutter
D. Bone nibbler
Correct Ans: D
Explanation

Answer- D. Bone nibbler

The GPC straight & curved bone nibbler (double action) is used for nibbling the bone.

The straight nibbler is used for general use and the curved nibbler is used for spinal surgery.


Question.16

Device shown in the image is used for what purpose?

AIIMS PG May-2019
A. Prevent viral infection
B. Prevent transfusion related reactions
C. Prevent infection
D. None
Correct Ans: B
Explanation

Answer- B. Prevent transfusion related reactions

It is a leuko- reduction filter which prevents transfusion related reactions


Question.17

What is the best way to manage the wound given in the image-

AIIMS PG May-2019
A. Cleaning & dressing
B. Cleaning & debridement
C. Use allograft
D. Use autograft
Correct Ans: A
Explanation

Answer- A. Cleaning & dressing


Question.18

Which is not associated with GDM (Gestational DM)?

AIIMS PG May-2019
A. Past history of fetal abnormality
B. Obesity
C. Hypertension
D. Macrosomia
Correct Ans: C
Explanation

Answer- C. Hypertension

A previous diagnosis of gestational diabetes or prediabetes, impaired glucose tolerance, or impaired fasting glycaemia.

Being overweight, obese

A previous pregnancy which resulted in a child with a macrosomia.


Question.19

False statement about Thyroglossal Duct cyst is-

AIIMS PG May-2019
A. Infected thyroglossal cyst from sinus
B. Lined by pseudostratified columnar epithelium
C. 40% cases have sub hyoid location
D. It is due to congenital
Correct Ans: A
Explanation

Answer- A. Infected thyroglossal cyst from sinus

Thyroglossal cysts are the most common cause of midline neck masses and are generally located caudal to (below) the hyoid bone.

Thyroglossal Duct Cysts are a birth defect.

A thyroglossal cyst is lined by pseudostratified, ciliated columnar epithelium while a thyroglossal fistula is lined by columnar epithelium.

A thyroglossal duct cyst may rupture unexpectedly, resulting in a draining sinus known as a thyroglossal fistula.


Question.20

Identify the disorder (undersurface of breast) as shown in the photograph below ?

AIIMS PG May-2019
A. Mondor’s disease
B. Paget’s disease
C. Mastitis
D. Chronic inflammatory abscess
Correct Ans: A
Explanation

Answer- A. Mondor’s disease

The condition shown in the picture above represents Mondor’s disease.

Mondor’s disease (also known as “Mondor’s syndrome of superficial thrombophlebitis”) is a rare condition which involves thrombophlebitis of the superficial veins of the breast and anterior chest wall. It sometimes occurs in the arm or penis. In axilla, this condition is known as axillary web syndrome.

Pediatrics

Question.1

A child can walk upstairs one step at a time, can ride cycle but can’t jump can also speak sentences, can tell his/her name gender but finds difficult to narrate the story.What is her actual developmental age

AIIMS PG May-2019
A. 1 yrs
B. 2
C. 3
D. 4
Correct Ans: B
Explanation

Ans. b) 2years

GROSS MOTOR MILESTONES:

Age Milestone
3 months Neck holding
5 months Rolls over
6months Sitting supported
8months Sitting without support
9months Stands with support
12months Stands without support, Walks but falls
15months Walks alone, Creeps upstairs
18months Runs, explores drawers
2 years Walks upstairs (baby steps), Jumps
3 years Walks upstairs (alternate feet), rides tricycle
4 years Hops on one foot, walks downstairs (alternate feet)

Question.2

Video based question

AIIMS NICU – neonate shown on O2 with nasal prongs, not intubated, sister shown inserting a tube through the mouth, camera zooms in, 18 (calibration) mark seen, the other (green) end as of now not connected to anything, video ends, what is the procedure that is being done?

VIDEO LINK: https://media.giphy.com/media/ln0aVCWqPvUtTWqBtT/giphy.gif

AIIMS PG May-2019
A. Oral suction
B. Oropharyngeal suction
C. Nasogastric tube insert
D. Orogastric tube insertion
Correct Ans: D
Explanation

Ans. D. Orogastric tube insertion


Question.3

Pediatric patient presented with 45 mins h/o continuous convulsions.

CASE 1: SR told to give iv lorazepam but J.R cant secured iv line. Then what he has given?

AIIMS PG May-2019
A. Rectal diazepam
B. Inhalation Phenobarbital
C. IV carbamazepine
D. Subcutaneous midazolam
Correct Ans: A
Explanation

Answer A. Rectal diazepam

  • Benzodiazepines are Drug of Choice and in pediatric patients rectal route should be preffered.

Reference https://www.uspharmacist.com/article/emergent-treatment-of-status-epilepticus-in-children


Question.4

CASE -2  SR visit again but the condition is not improved but this time IV cannula was set. What drug should be given now?

AIIMS PG May-2019
A. Midazolam
B. I/V Phenobarbital
C. Oral valproate
D. IV carbamazepine
Correct Ans: B
Explanation

Ans. In this case the First Line has failed . So for second line therapy I/V Phenobarbital is preffered

Reference – https://www.uspharmacist.com/article/emergent-treatment-of-status-epilepticus-in-children


Question.5

Sequential arrangement of fetal scans

AIIMS PG May-2019
A. Growth scan
B. Triple marker
C. Anomalous Scan
D. NT scan
Correct Ans: All:Growth scan, Triple marker, Anomalous Scan, NT scan
Explanation

Ans: d,b,c,a….

The NT scan must be done between 11 and 14 weeks pregnant, because this is when the base of baby’s neck is still transparent. (The last day for scan is  13 weeks and 6 days pregnant.)

Triple marker test is performed in pregnant women at the end of first trimester and the beginning of the second trimester.

The anomaly scan, also sometimes called the anatomy scan, 20 week ultrasound, or level 2 ultrasound, is a pregnancy ultrasound performed between 18–22 weeks

A growth scan is an ultrasound scan that determines whether your baby’s growth is normal. Doctors typically recommend it for women during the third trimester of pregnancy; one of the reasons it is also a fetal growth scan between 28 weeks and 32 weeks of pregnancy.


Question.6

4yr boy absence of right testes, diagnostic laparoscopy done, a blind end vessel seen what to do next?

AIIMS PG May-2019
A. Open laprotomy
B. Nothing to be done
C. Scrotal approach
D. Inguinal approach
Correct Ans: D
Explanation

Ans. D. Inguinal approach

  • Cryptorchidism is the most common congenital abnormality of the genitourinary tract.
  • Cryptorchidism means hidden testis.
  • An absent testis may be due to agenesis or atrophy secondary to intrauterine vascular compromise also known as the “vanishing testis syndrome”.
  • Bilaterally absent testes is anorchia which is 10% cases.
  • More common on Right Side.
  • Complications of undescended testes
  • Torsion can be seen in incomplete testicular descent
  • Sterility is seen in bilateral cases (especially intra-abdominal testes)
  • Incomplete testicular descent predisposes to malignant disease; cancer is more common in an incompletely descended testes-orchidopexy may or may not diminish the risk.
  • Atrophy of an inguinal testes before puberty may possibly be caused by recurrent minor trauma.
  • In patients with a unilateral nonpalpable testis, a descended testis that is larger than expected suggests an atrophic undescended testis; confirmation requires surgical intervention typically via diagnostic laparoscopy to seek an intra-abdominal testis or confirm testicular agenesis. However, scrotal or inguinal exploration is sometimes done if a testicular remnant distal to the internal inguinal ring is suspected.

Question.7

A 4 year-old boy, develops a large erythematous rash around the site of a mosquito bite. One month later, he is taken to a pediatrician because of a puffy face and swollen ankles. The scanty urine sample has a reddish-brown hue, and contains both red blood cells and protein. Which of the following distinctive features would be most likely to be seen on renal biopsy?

AIIMS PG May-2019
A. Fusion of podocyte foot processes
B. IgA in the mesangium
C. Linear IgG deposits
D. Subepithelial electron dense humps
Correct Ans: D
Explanation

Ans. D. Subepithelial electron dense humps

The disease is poststreptococcal glomerulonephritis, which can follow either streptococcal pharyngitis (one to two weeks after the infection) or skin infection (three to six weeks after the infection).

The child is showing signs of both nephritis (hematuria) and nephrosis (puffy face and swollen ankles).

The characteristic feature of this disease on renal biopsy is the presence of subepithelial humps, visible either by light or electron microscopy.

Fusion of podocyte foot processes suggests minimal change disease.

IgA in the mesangium suggests Berger’s disease.

Linear IgG deposits suggests anti-glomerular basement membrane disease, which is called Goodpasture’s disease if it is accompanied by pulmonary damage.

Onion-skinning of renal arterioles suggests malignant hypertension.


Question.8

A boy presented with a history of fever, headache and vomiting since 3 days and disorientation since I day. On examination, neck rigidity was positive. He also had 1 episode of generalized tonic clonic seizures (GTC) and then he became unconscious. CECT was found to be normal. CSF examination reveals a cell count of 300 cells/mm3 (polymorphs: 50-70%), protein 70 mg/dL, sugar 50 mg/dL (blood: 95 mg/dL). Which of the following is the most likely diagnosis?

AIIMS PG May-2019
A. Pyogenic meningitis
B. Tubercular meningitis
C. Herpes encephalitis
D. Cerebral malaria
Correct Ans: A
Explanation

Ans. a. Pyogenic meningitis (ReJ: Harrison l9/e p887, 891, llll, l8/e p1029, 1692, 3414)

The classic CSF abnormalities in bacterial meningitis are:

  • Elevated opening pressure (> 180 mm H2O in 90%)
  • Polymorphonuclear leakocytosis (>100 cells/ul in 90%)
  • Decreased glucose concentration (
  • Elevated protein concentration (>45 mg/dl)

Question.9

Arrange  the following in the sequence of closure :

AIIMS PG May-2019
A. Umbilical Artery
B. Umbilical Vein
C. Ductus venosus
D. Foramen ovale
E. Ductus arteriosus
Correct Ans: All:Umbilical Artery, Umbilical Vein, Ductus venosus, Foramen ovale, Ductus arteriosus
Explanation

Ans.E>D>C>A>B

Within a week of birth, the neonate’s umbilical vein is completely obliterated and is replaced by a fibrous cord called the round ligament of the liver (also called ligamentum teres hepatis).

Closure of the umbilical vein usually occurs after the umbilical arteries have closed

Obs / Gyne

Question.1

Arrange the following steps in sequence as it occurs in spermatogenesis.

AIIMS PG May-2019
A. Spermatocyte
B. Spermatogonia
C. Spermatozoa
D. Spermatid
Correct Ans: All:Spermatocyte, Spermatogonia, Spermatozoa, Spermatid
Explanation

Ans-b)Spermatogonium> a)Spermatocyte >d)Spermatid >c)Spermatozoa


Question.2

Chicken pox infection in mother most dangerous for the baby when it occurs at

AIIMS PG May-2019
A. 14-28 weeks
B. 3 -9 weeks
C. 28-37weeks
D. 8-14weeks
Correct Ans: C
Explanation

Ans. C. 28-37weeks

  • Before 28 weeks pregnant: there’s no evidence you are at increased risk of suffering a miscarriage. However, there’s a small risk baby could develop foetal varicella syndrome (FVS). FVS can damage the baby’s skin, eyes, legs, arms, brain, bladder or bowel.
  • Between weeks 28 and 36 of pregnancy: the virus stays in the baby’s body but doesn’t cause any symptoms. However, it may become active again in the first few years of the baby’s life, causing shingles.
  • After 36 weeks of pregnancy:  baby may be infected and could be born with chickenpox.

Question.3

All are used in PPH expert :

AIIMS PG May-2019
A. Mife pristine
B. Carboprost
C. Misoprostol
D. Ergometrine
Correct Ans: A
Explanation

Ans. A. Mife pristone


Question.4

Which of the following can be used to delay preterm contractions of uterus with best neurological outcome for the fetus?

AIIMS PG May-2019
A. Mgso4
B. Nifidipine
C. Ritodrine
D. Isoxprine
Correct Ans: A
Explanation

Ans. A.MgSo4

Antenatal magnesium sulfate for both tocolysis and fetal neuroprotection in premature rupture of the membranes before 32 weeks’ gestation.

Other tocolytic drugs:

  • Ritodrine, salbutamol and magnesium sulphate are tocolytic drugs used to terminate preterm labour and delivery.
  • Other tocolytic drugs are isoxsuprine, indomethacin, calcium channel blockers, glyceryl trinitrate, atosiban and glyceryl trinitrate.

Question.5

Assertion reason type q:

A: Hot flushes are experienced by women during menopause

R:Hot flushes are due to withdrawal or fluctuation of estrogen.

AIIMS PG May-2019
A. Both assertion and reason are true and reason is the correct explanation of assertion
B. Both assertion and reason are true but reason is not the correct explanation of assertion
C. Assertion is true but reason is false
D. Reason in true but assertion is false
Correct Ans: A
Explanation

Ans.A. Both assertion and reason are true and reason is the correct explanation of assertion

  • Hot Flushes : They are the ‘hallmark’ of menopause. Hot flushes are described as recurrent transient period of flushing, sweating and a sensation of heat often accompanied by palpitations, feelings of anxiety and sometimes followed by chills.
  • The entire episode lasts no more than 1-3 minutes and recurs 5-10 times / day (can occur upto 30 times a day).
  • Short term estrogen therapy results in resolution of hot flushes.

Question.6

Best Age to start bone mineral density test in female.

AIIMS PG May-2019
A. After 50 years
B. After 55 years
C. After 60 years
D. After 65 years
Correct Ans: D
Explanation

Ans. d. After 65 years

NOF recommends:

Bone density test :

  • In woman age 65 or older.
  • In man age 70 or older.

Question.7

A 25 year old nulliparous woman at 35 weeks’ gestation comes to the labor and delivery ward complaining of contractions, a headache, and flashes of light in front of her eyes. Her pregnancy has been uncomplicated except for an episode of first trimester bleeding that completely resolved. She has no medical problems. Her temperature is 37 C (98.6 F), blood pressure is 160/110 mm Hg, pulse is 88/minute, and respirations are 12/minute. Examination shows that her cervix is 2 centimeters dilated and 75% effaced, and that she is contracting every 2 minutes. The fetal heart tracing is in the 140s and reactive. Urinalysis shows 3+ proteinuria. Laboratory values are as follows: leukocytes 9,400/mm3, hematocrit 35%, platelets 101,000/mm3. Aspartate aminotransferase (AST) is 200 U/L, and ALT 300 U/L. Which of the following is the most appropriate next step in management?

AIIMS PG May-2019
A. Administer oxytocin
B. Discharge the patient
C. Encourage ambulation
D. Start magnesium sulphate
Correct Ans: D
Explanation

Ans. D. Start magnesium sulphate

  • This patient has severe preeclampsia. Preeclampsia is diagnosed on the basis of hypertension, edema, and proteinuria.
  • Severe preeclampsia may be diagnosed when the patient has one of the following: a headache that does not respond to analgesics, visual changes, seizure, very elevated blood pressures, pulmonary edema, elevated liver function tests, severe proteinuria, oliguria, an elevated creatinine, thrombocytopenia, hemolysis, intrauterine growth restriction, or oligohydramnios.
  • The management of severe preeclampsia after 32 weeks is with delivery.
  • Prior to 32 weeks, consideration may be given to expectant management of the patient depending on the clinical circumstances. This patient is at 35 weeks’ with headache, visual changes, elevated blood pressures, thrombocytopenia, and elevated liver function tests. She, therefore, needs to be delivered.
  • She appears to already be in labor as she is contracting every two minutes and her cervix is dilated and effaced.
  • At this point, magnesium sulfate should be started.
  • Magnesium sulfate has consistently been demonstrated to be the most effective medication for seizure prophylaxis in women with preeclampsia.
  • To administer oxytocin would not be necessary for this patient. She appears to already be in labor with contractions every two minutes.
  • To discharge the patient would absolutely be incorrect. Severe preeclampsia need to be delivered or, at the very least, admitted to the hospital. There is no role for discharging a patient home in the management of severe preeclampsia.
  • To encourage ambulation would also be incorrect. Severe preeclampsia should be kept on bed rest.

Ref:  Cunningham F.G., Leveno K.J., Bloom S.L., Hauth J.C., Rouse D.J., Spong C.Y. (2010). Chapter 34. Pregnancy Hypertension. In F.G. Cunningham, K.J. Leveno, S.L. Bloom, J.C. Hauth, D.J. Rouse, C.Y. Spong (Eds), Williams Obstetrics, 23e.


Question.8

P3L3 came to opd with postcoital bleeding via and pap positive p/v cervix hypertrophied bleed on touch  diagnosis

AIIMS PG May-2019
A. CA cervix
B. Fibroid
C. Cervicitis
D. Cervical polyp
Correct Ans: A
Explanation

Ans. A. CA cervix

SYMPTOMS:

  • Arises from: Squamo-columnar junction
  • Earliest symptom: Post-coital bleeding

As the cancer progresses, symptoms may include:

  • Unusual vaginal discharge
  • Vaginal bleeding between periods
  • Bleeding after menopause
  • Pyometra
  • Bleeding or pain during sex
  • MC site: Ectocervix
  • Lymph nodes affected: Obturator, hypogastric and external iliac
  • Time taken for conversion of CIN to invasive Ca: 10 years
  • MC type: Squamous cell Ca
  • 100% cure rates are seen in: CIS
  • Uremia: altered sensorium and is having hiccups
  • MC cause of death: Renal failure
  • Ca cervix can be prevented by screening

Both positive Pap smear and VIA test is suggestive of CA cervix


Question.9

A 35 year old female presented with post coital bleeding. Next step of management is:

AIIMS PG May-2019
A. Pap smear and colposcopy
B. Visual inspection with acetowhite
C. Visual inspection with lugol’s iodine
D. Colposcopy directed biopsy
Correct Ans: A
Explanation

Ans. A. Pap smear and colposcopy

  • Postcoital bleeding (PCB) has many causes, including vaginitis, cervicitis, cervical dysplasia and malignancy, uterine lesions, and pregnancy.
  • Some women with postcoital bleeding may have pathologic lesions identified at colposcopic evaluation that had been missed by Pap smear screening.
  • Thus, colposcopic examination is considered for women with unexplained postcoital bleeding.

Ref: Hoffman B.L., Schorge J.O., Schaffer J.I., Halvorson L.M., Bradshaw K.D., Cunningham F.G., Calver L.E. (2012). Chapter 8. Abnormal Uterine Bleeding. In B.L. Hoffman, J.O. Schorge, J.I. Schaffer, L.M. Halvorson, K.D. Bradshaw, F.G. Cunningham, L.E. Calver (Eds), Williams Gynecology, 2e.


Question.10

28 yr with infertility lapro tube uterus healthy ovary diagnosis

AIIMS PG May-2019
A. PCOS
B. Ovarian cyst
C. Fibroid
D. Endometriosis
Correct Ans: D
Explanation

Ans.D. Endometriosis

  • In endometriosis, cause of infertility is
  • Immobility of tubes
  • Anovulation
  • Tubal block
  • Male factor: 30%
  • Tubal, uterine & peritoneal factor: 25%
  • Ovarian factor: 25%
  • Cervical factor: 10%
  • Unexplained factor: 10%

Question.11

Fetal ECHO shows congenital heart block, what should be the mother screened for?

AIIMS PG May-2019
A. SLE
B. Myxoma
C. APLA
D. None
Correct Ans: A
Explanation

Ans. a). SLE

Congenital heart block is a rare disorder that occurs in about one out of 22,000 live births.

In most cases, the cause is not known, but babies of mothers with lupus or other autoimmune diseases, or babies with congenital heart disorders, are at higher risk.

Pregnant women who have autoimmune diseases, such as lupus or Sjogren’s syndrome, are at an increased risk of having a baby with congenital heart block.

Also, a tumor on the baby’s heart can cause heart block.


Question.12

A 49 year old female was prescribed hormone replacement therapy (HRT). HRT is useful in all of the following, EXCEPT:

AIIMS PG May-2019
A. Flushing
B. Osteoporosis
C. Vaginal atrophy
D. Coronary heart disease
Correct Ans: D
Explanation

Ans. D. Coronary heart disease

Benefits and Risks of Postmenopausal Hormone Therapy (HT): 

Definite Benefits

  • Symptoms of menopause (e.g, Flushing, vaginal atrophy)
  • Osteoporosis

Definite Risks

  • Endometrial cancer
  • Venous thromboembolism
  • Breast cancer
  • Gallbladder disease

Probable or Uncertain Risks and Benefits

  • Coronary heart disease
  • Stroke
  • Ovarian cancer
  • Colorectal cancer
  • Diabetes mellitus
  • Cognitive dysfunction

Ref: Manson J.E., Bassuk S.S. (2012). Chapter 348. The Menopause Transition and Postmenopausal Hormone Therapy. In Longo D.L., Fauci A.S., Kasper D.L., Hauser S.L., Jameson J, Loscalzo J (Eds), Harrison’s Principles of Internal Medicine, 18e.

Orthopedics

Question.1

Match the following-

  1. Pilons fracture- 1. Trimalleolar #
  2. Chauffeur fracture- 2. fracture of the distal part of the tibia
  3. Cotton fracture-  3. fracture of radial styloid process.
  4. Runner fracture- 4. Lateral malleolar #
AIIMS PG May-2019
A. A-2, b-3, c-4, d-1
B. A-3, b-4, c-2, d-1
C. A-1, b-2, c-3, d-4
D. A-2, b-4, c-3, d-1
Correct Ans: A
Explanation

Answer- A. A-2, b-3, c-4, d-1

  • Pilons – fracture of the distal part of the tibia
  • Chauffeur- fracture of radial styloid process.
  • cotton- Lateral malleolar #
  • Runners fracture- Trimalleolar #

Question.2

Image showing x-ray of osteolytic lesion on lateral condyle of femur. What will be the line of treatment?

AIIMS PG May-2019
A. extended curettage with bone autograft
B. extended curettage with bone allograft
C. curettage only
D. Bone biopsy
Correct Ans: D
Explanation

Answer- D. Bone biopsy


Question.3

Specific for Ankylosing spondylitis is true?

AIIMS PG May-2019
A. HLA
B. B27
C. Sacroileitis
D. Raised ESR
Correct Ans: C
Explanation

Answer- C. Sacroileitis


Question.4

Scaphoid fracture at waist with retrograde blood supply. Which segment is most susceptible to avascular necrosis?

AIIMS PG May-2019
A. Proximal
B. Distal
C. Middle
D. Scaphoid tubercle
Correct Ans: A
Explanation

Answer- A. Proximal

  • Most common site of scaphoid fracture is Waist.
  • Fractures can occur essentially anywhere along the scaphoid, but distribution is not even:
  1. waist of scaphoid: 70-80%
  2. proximal pole: 20%
  3. distal pole (or so-called scaphoid tubercle): 10%


Question.5

Which of the following is true about anterior shoulder dislocation –

AIIMS PG May-2019
A. It is the most common type of shoulder dislocation
B. It is most commonly subclavicular
C. Patient keeps his arm in saluting position
D. Injury to brachial plexus may occur
Correct Ans: A
Explanation

Answer- A.  It is the most common type of shoulder dislocation

Most common type of shoulder dislocation is anterior dislocation (subcoracoid being most common).

Patient keeps his arm by the side of the body in  a position of abduction and internal rotation.


%d bloggers like this:
Malcare WordPress Security